Sei sulla pagina 1di 50

CASE FACTS ISSUE HELD CONSTI ISSUE

Valmonte v. General de Case Book W/N checkpoints, which are No one can be compelled to share with the Section 2 does not prohibit every search
Villa - 178 SCRA 211, designed as a security government its ideological beliefs and and seizure but only “unreasonable
(MAIN) and 185 SCRA 665 Petitioners have filed the instant motion and measure to entrap criminals practices, or commed its political, social, searches and seizures.”
supplemental motion for reconsideration of the and insurgents and to and economic policies or performance. But
Nat Sept. 29, 1989 Court’s decision (dismissal of the constitute a dragnet for all one must concede to it the basic right to Check points are not illegal per se. Under
petitioner’s petition seeking for the declaration of types of articles in illegal defend itself from its enemies, and to exceptional circumstances, as where the
checkpoints as unconstitutional and their trade, is violative of the pursue its programs of government survival of organized government is on the
dismantling and/or banning) Constitution. intended for public welfare; and in pursuit balance, or where the lives and safety of
of those objectives, the government has people are in grave peril, checkpoints may
Check points are not illegal per se. Under equal right, under its police power, to select be allowed and installed by the
exceptional circumstances, as where the survival of the reasonable means and methods best government. Implicit in this proposition that
organized government is on the balance, or where for achieving them. The checkpoint is when the perils are removed, checkpoints
the lives and safety of people are in grave peril, evidently one of such means chosen. have no reason to remain.
checkpoints may be allowed and installed by the
government. Implicit in this proposition that when Although the routine checkpoint stop does
the perils are removed, checkpoints have no reason intrude to a certain extent on the motorist’s
to remain. right to “free passage without interruption,”
but it cannot be denied that it involves only
Recent and on-going events have pointed to the a brief detention of travellers during which
continuing validity and need for checkpoints the vehicle’s occupants are required to
manned by either military or polic forces. There was answer a brief question or two. For as long
a 6th attempted coup d’etat, stronger than all the as the vehicle is neither searched nor its
previous ones, was staged last Dec. 1, 1989. The occupants subjected to a body search, and
NPA, through its sparrow units, has not relented but the inspection of the vehicle is limited to a
instead accelerated its liquidation of armed forces visual search, said routine check cannot be
and police personnel. Murders, sex crimes, hold-ups regarded as violative of an individual's right
and drug abuse have become a daily occurences. against unreasonable search.
Unlicensed firearms and ammunition have become
favorite objects of trade. Smuggling is at an all-time Routine checks when conducted in fixed
high. areas are less intrusive. As proven by the
U.S. Supreme Court: Routine checkpoint do
not intrude on the motoring public. First,
the potential interference with traffic is
minimal; Second, checkpoint operations are
less discretionary.

Vehicles are generally allowed to pass after


the routine inspection and a few questions.
If the vehicle is stopped and extensively
searched, it is due to some probable cause
which justifies a reasonable belief of the
men at the checkpoint that either the
motorist is a law-offender or the contents of
the vehicle are or have been instruments of
some offense.

These warrantless searches may be


compared to: searches and seizures
accompanying warrantless arrests during
the commission of a crime or immediately
thereafter, or warrantless searches of
incoming or ongoing passengers at the
arrivals and departure areas of an
international airport.

The court’s decision on checkpoints does


not, in any way, validate nor condone
abuses committed by the military manning
the checkpoints. The court must assume
that the men in uniform live and act by the
code of honor and they are assigned to the
checkpoints to protect, and not abuse, the
citizenry. (The checkpoints are being
blamed for the increase of the prices of
foodstuffs entering Manila due to the
abuses of those manning these points.)

When abuse marks the operation of a


checkpoint, the citizen is not helpless. The
military is not above but subject to the law.
And the courts exist to see that the law is
supreme. Soldiers, including those who
man the checkpoints, who abuse their
authority act beyond the scope of their
authority and are, therefore, liable
criminally and civilly for their abusive acts.

The motion and supplemental Motion for


Reconsideration are DENIED.

SJS v. DDB 570 SCRA 410 Case book - but it is around 10 pages so I tried to Consolidated Issues: 1. (Pimentel’s Contention) Section Sec 36. Authorized Drug Testing –
consolidate it here :) 36(g) of RA 9165 and COMELEC Authorized Drug testing shall be done by
RC 1. Do Section 36(g) Resolution 6486 are any government forensic laboratories or by
This is a consolidated case of 3 petitioners (1. of RA 9165 and UNCONSTITUTIONAL. any of the drug testing laboratories
Pimentel, 2. SJS and 3. Lacerna) assailing the COMELEC accredited and monitored by the DOH to
constitutionality of Sec 36 of RA 9165 – Dangerous Resolution No It violates the Constitution Article 6 Sec safeguard the quality of the test results. Xxx
Drugs Act of 2006 and Resolution 6486 (Issued by 6486 impose 3, imposing additional qualification to The drug testing shall employ, among
the COMELEC – this is under Pimentel’s contention). additional candidates which should only be limited others, two (2) testing methods, the
qualification for to the ones expressly mentioned. screening test which will determine the
candidates for positive result as well as the type of drug
the Comprehensive Dangerous Drugs Act of 2002, senator? The Constitution is the basic law to used and the confirmatory test which will
insofar as it requires mandatory drug testing of (1) Corollarily, can which all laws must conform; no act shall confirm a positive screening test. Xxx The
candidates for public office; (2) students of Congress enact a be valid if it conflicts with the following shall be subject to undergo drug
secondary and tertiary schools; (3) officers and law prescribing Constitution. In the discharge of their testing:
employees of public and private offices; and (4) qualifications for defined functions, the three
persons charged before the prosecutor’s office of a candidates for departments of government have no xxx
crime with an imposable penalty of imprisonment senator in choice but to yield obedience to the c. Students of secondary and tertiary
of not less than 6 years and 1 day. addition to those commands of the Constitution. schools
laid down by the Whatever limits it imposes must be
Pimentel Assailed (1-a) - public office Constitution? observed. d. Officers and employees of public and
SJS Assailed (2 - c), (3 - d) and (4 -f) private offices – whether domestic or
Lacerna Assailed (2 - c) and (3 - d) 2. Are paragraphs overseas, shall be subject to undergo a
(c),(d), (f) and (g) 2. (SJS and Lacerna’s Contentions) random drug test as contained in the
Placed Sec 36 in the Consti Issue Tab. of Sec. 36 RA 9165 Sec 36 (c) and (d) of RA 9165 are company’s work rules and regulations, xxx,
unconstitutional? CONSTITUTIONAL. for the purposes of reducing the risk in the
Specifically, do workplace. Any officer or employee found
these paragraphs For (c) It cited Vernonia and reasoned positive for use of dangerous drugs shall be
violate the right to that the provisions of RA 9165 requiring dealt with administratively which shall be a
privacy, the right mandatory, random, and suspicionless ground for suspension or termination,
against drug testing of students are subject to the provisions of Article 282 of
unreasonable constitutional. Indeed, it is within the the Labor Code and pertinent provisions of
searches and prerogative of educational institutions to the Civil Service Law;
seizures and the require, as a condition for admission,
equal protection? compliance with reasonable school rules f. All persons charged before the
Or do they and regulations and policies. To be sure, prosecutor’s office with a criminal offense
constitute undue the right to enrol is not absolute; it is having an imposable penalty of
delegation of subject to fair, reasonable, and equitable imprisonment of not less than 6 months
legislative power? requirements. and 1 day shall undergo a mandatory drug
test.
For (d)
As the warrantless clause of Sec. 2, Art III g. All candidates for public office whether
of the Constitution is couched and as has appointed or elected both in the national or
been held, “reasonableness” is the local government shall undergo a
touchstone of the validity of a mandatory drug test.
government search or intrusion. And
whether a search at issue hews to the Resolution 6486 - requires candidates for
reasonableness standard is judged by the public office to have a mandatory drug
balancing of the government-mandated testing
intrusion on the individual's privacy
interest against the promotion of some Vernoia Doctrine:
compelling state interest. In the criminal 1) schools and their administrators stand in
context, reasonableness requires loco parentis with respect to their students;
showing of probable cause to be
personally determined by a judge. Given (2) minor students have contextually fewer
that the drug-testing policy for
employees—and students for that rights than an adult, and are subject to the
matter—under RA 9165 is in the nature custody and supervision of their parents,
of administrative search needing what guardians, and schools;
was referred to in Vernonia as “swift and (3) schools, acting in loco parentis, have a
informal disciplinary procedures,” the duty to safeguard the health and well-being
probable-cause standard is not required of their students and may adopt such
or even practicable. Be that as it may, measures as may reasonably be necessary
the review should focus on the to discharge such duty; and
reasonableness of the challenged
administrative search in question. (4) schools have the right to impose
conditions on applicants for admission that
While Sec 36 (f) is Declared are fair, just, and non-discriminatory.
UNCONSTITUTIONAL.

Unlike the situation covered by Sec.


36(c) and (d) of RA 9165, the Court finds
no valid justification for mandatory
drug testing for persons accused of
crimes. In the case of students, the
constitutional viability of the
mandatory, random, and suspicionless
drug testing for students emanates
primarily from the waiver by the
students of their right to privacy when
they seek entry to the school, and from
their voluntarily submitting their
persons to the parental authority of
school authorities.

People v. Veloso - 48 PHIL. May, 1923, the building located at No. 124 Calle W/N the search warrant and Yes, in the first place, the affidavit for the
Philippine Code on Criminal Procedure
169 Arzobispo, City of Manila, was used by an arrest was valid. search warrant and the search warrant provides that “a search warrant shall not
organization known as the Parliamentary Club. Jose itself described the building to be searched issue except for probable cause and upon
Bryce Ma. Veloso was at that time a member of the House as "the building No. 124 Calle Arzobispo, application supported by oath particularly
of Representative of the Philippine Legislature. He City of Manila, Philippine Islands." This, describing the place to be searched and the
was also the manager of the club. without doubt, was a sufficient designation person of thing to be seized.”
of the premises to be searched. The name and description of the accused
The police of Manila had reliable information that should be inserted in the body of the
the so-called Parliamentary Club was nothing more As the search warrant stated that John Doe warrant and where the name is unknown
than a gambling house. Indeed, on May 19, 1923, J. had gambling apparatus in his possession in there must be such a description of the
F. Townsend, the chief of the gambling squad, had the building occupied by him at No. 124 person accused as will enable the officer to
been to the club and verified this fact. As a result, Calle Arzobispo, City of Manila, and as this identify him when found.
on May 25, 1923, Detective Andres Geronimo of the John Doe was Jose Ma. Veloso, the manager
secret service of the City of Manila, applied for, and of the club, the police could identify John A warrant for the apprehension of a person
obtained a search warrant from Judge Garduño of Doe as Jose Ma. Veloso without difficulty. whose true name is unknown, by the name
the municipal court. Thus, the police attempted to of "John Doe" or "Richard Roe," "whose
raid the Parliamentary Club a little after three in the other or true name in unknown," is void,
afternoon. They found the doors to the premises without other and further descriptions of
closed and barred. Accordingly, one band of police the person to be apprehended, and such
including policeman Rosacker, ascended a warrant will not justify the officer in acting
telephone pole, so as to enter a window of the under it. Such a warrant must, in addition,
house. Other policemen, headed by Townsend, contain the best description personae
broke in the outer door. possible to be obtained of the person or
persons to be apprehended, and this
Once inside the Parliamentary Club, nearly fifty description must be sufficient to indicate
persons were apprehended by the police. One of clearly the proper person or persons upon
them was the defendant Veloso. Veloso asked whom the warrant is to be served; and
Townsend what he wanted, and the latter showed should state his personal appearance and
him the search warrant. Veloso read it and told peculiarities, give his occupation and place
Townsend that he was Representative Veloso and of residence, and any other circumstances
not John Doe, and that the police had no right to by means of which he can be identified.
search the house. Townsend answered that Veloso
was considered as John Doe. As Veloso's pocket was
bulging, as if it contained gambling utensils,
Townsend required Veloso to show him the
evidence of the game. About five minutes was
consumed in conversation between the policemen
and the accused the policemen insisting on
searching Veloso, and Veloso insisting in his refusal
to submit to the search.

At last the patience of the officers was exhausted.


So policeman Rosacker took hold of Veloso only to
meet with his resistance. Veloso bit Rosacker in the
right forearm, and gave him a blow in another part
of the body, which injured the policeman quite
severely. Through the combined efforts of
Townsend and Rosacker, Veloso was finally laid
down on the floor, and long sheets of paper, of
reglas de monte, cards, cardboards, and chips were
taken from his pockets.

All of the persons arrested were searched and then


conducted to the patrol wagons. Veloso again
refused to obey and shouted curses against the
police department. It was necessary for the
policemen to conduct him downstairs. At the door,
Veloso resisted so tenaciously that three policemen
were needed to place him in the patrol wagon.

Alvarez v. CFI - 64 PHIL. 33 - Petitioner asks that warrant of June 3, 1936, issued W/N search and seizure NO. Warrant was not valid Search warrant is an order in writing issued
by CFI-Tayabas, ordering the search and seizure of warrant was valid in the name of the People of the Philippine
Robynne certain accounting documents at any time of day - While power of search and seizure is Islands, signed by a judge or justice of the
and night as well as the order authorizing the necessary to the public welfare, it must not peace, and directed to a peace officer,
agents of the Anti-Usury Board to retain the articles violate the consti rights of the citizens commanding him to search for personal
seized be declared illegal and set aside and the property and bring it to court
return of the items to him. - The gereral rule is that statutes
authorizing search and seizure must be General Order No. 58 (GO) sec 97: a search
- Petitioner contends that the search warrant issued strictly construed warrant shall not issue except for probable
is illegal because it was based upon the affidavit of cause and upon application supported by
Agent Almeda who declared in his oath that he had - There must be probable cause as well as oath particularly describing the place to be
no personal knowledge of that facts that served as an application supported by oath of the searched and the person/thing to be seized
basis for the issuance of the warrant but he knew of applicant and witness he may produce.
them through mere information by a reliable source Oath must be based on personal knowledge True test of sufficiency if an affidavit is
of the facts because the purpose of it is to whether is was drawn in a manner that
- Anti-Usury Board insinuates that petitioner cant convince the magistrate of the existence of perjury could be committed
question the validity of the search or proceedings probable cause
because he waived his Consti rights in proposing a General Code No 58, sec 98: judge/justice
compromise where latter would pay a fine. What constitutes unreasonable search and of peace must examine under oath the
seizure is a judicial question: includes complainant and witnesses he may produce
purpose, presence/absence of probable and take depositions in writing
cause, manner search was made, character
of articles, etc. Article 3, section 1, paragraph 3 of Consti
prohibits unreasonable search and seizure.
- Neither GO or Consti provide that it is
imperative to take the depositions of the GO 58, sec 101: search may be made at
witnesses in addition to the affidavit night when it is positively asserted in the
affidavit that property is on the person or
- If affidavit is sufficient, other witnesses place ordered to be searched.
may be dispensed with. If not affidavit is
based on hearsay, judge may require
affidavit of more witnesses.

- The search warrant and subsequent


seizure are illegal because: agent had no
personal knowledge of the facts, search
cant be done at night, warrant was issued
for the seizure of items w/c will be used in
a criminal proceeding against petitioner, it
didnt appear that article seized were in the
possession of the petitioner

- As to the waiver, there was no such thing


because petitioner denied offer of
compromise and that said compromise
referred, if there was one, to the institution
of the crim proceedings and not to the
search warrant.
- Appeal would not be a plain, speedy and
adequate remedy because a long time
would elapse before petitioner would
recover his property.

Stonehill v. Diokno – 20 From the Case Book: Whether or not the search NO. The Court ruled that the warrants TWO (2) POINTS THAT MUST BE STRESSED
SCRA 383 warrant was valid and sanctioned the seizure of all records of the IN THE CONSTITUTIONAL MANDATE OF
Upon the application of the officers of the therefore the things seized petitioners and the corporations, whatever THE RIGHT AGAINST UNREASONABLE
Mar government, several Judges issued, on different may be admitted as their nature, thus openly contravening the SEARCHES AND SEIZURES:
dates, a total of 42 search warrants against the evidence. explicit command of our Bill of RIghts -- a) that no warrant shall issue but
Petitioners and/or the corporations of which they that the things to be seized be upon probable cause, to be
were officers. The search warrant enumerated the PARTICULARLY DESCRIBED -- as well as determined by the judge in the
following to be seized: books of accounts, financial tending to defeat its major objective: THE manner set for in the said
records, vouchers, correspondence, receipts, ELIMINATION OF GENERAL WARRANTS. provision; and
ledgers, journals, typewriters, and other documents b) that the warrant shall particularly
and/or papers showing all business transactions The Court split into two (2) major groups describe the things to be seized.
including disbursements receipts, balance sheets, the documents, papers, and things seized
and profit and loss statements and Bobbins. under the alleged authority of the warrant MAJOR OBJECTIVE: to eliminate general
in question: warrants
The application also has for the subject of the a) those found and seized in the
offense: “those stolen or embezzled and proceeds or office of the corporations; and
fruits of the offense, or used or intended to be used b) those found and seized in the
as the means of committing the offense which is a residence of the petitioners.
violation of Central Bank Laws, Tariff and Customs As regards the first group, the Court held
Laws, Internal Revenue Code, and Revised Penal that petitioners may not validly object to
Code.” the use in evidence against them of the
documents, papers, and things seized from
Petitioners allege that such search warrant was null the offices and premises of the corporation
and void for the following reasons: because they have no cause of action to
a) they do not describe with particularity the assail the legality of the contested warrants.
documents, books, and things to be Corporations have their respective
seized; personalities, separate and distinct from
b) cash money, not mentioned in the the personality of the petitioners regardless
warrants, were actually seized; the amount of shares of stock or of the
c) the warrants were issued to fish evidence interest of each in the corporation.
against the petitioners in deportation
cases filed against them; On the other hand, those seized from their
d) the searches and seizures were made in respective residences is a different story.
an illegal manner; and The Court did not uphold its validity to such
e) the documents, papers, and cash money for no specific offense has been alleged in
seized were not delivered to the Courts the said applications and as such it would
that issued the warrants, to be disposed have been impossible for the judges who
of in accordance with the law. issued the warrants to have found
probable cause. FURTHERMORE, TO
UPHOLD ITS VALIDITY WOULD PLACE THE
SANCTITY OF THE DOMICILE AND THE
PRIVACY OF THE COMMUNICATION AND
CORRESPONDENCE AT THE MERCY OF THE
WHIMS, CAPRICE OR PASSION OF PEACE
OFFICERS.

Central Bank v. Morfe Central Bank (CB) announced that, in line with RA W/N failure of the witness No. No victims named is immaterial for this If the questioned/ illegal transactions
337, all organizations including Savings and Loans to mention particular case. involves a general pattern of business of the
Jill Associations need to be authorized by the individuals/victims proves The records suggest that the illegal organization, the identification of victims
Monetary Board (MB) to accept deposits from the that he has no personal transactions constituted the GENERAL and specific transactions is not required.
public, and engage in the Banking Activities. knowledge of illegal PATTERN OF BUSINESS of the organization.
activities, making the first (this can be seen in the By-laws of FMS: “to But if the transactions involve Isolated
Governor of CB directed investigations to make sure search warrant extend financial assistance, in the form of Transactions, then the identification of
that all such organizations comply with RA 337. unreasonable. loans, to its members, with funds deposited actually injured parties/ victims is
with them.” necessary.
The intelligence department of CB filed for a search
warrant to search First Mutual Savings and Loan In this case, the existence of an actual
Organization (FMS). The officer said that through victim, or for someone to be actually
observation and investigation, they found that FMS injured is not necessary before a warrant
was engaged in illegal banking activities (because may be issued. Because, the law seeks to
they receive deposits for safekeeping/ protect the public not only from Actual
disbursement from its members, w/o complying Injury, but also Potential Injury.
with requisites of RA 337, specifically w/o getting
authorization from monetary board). The identification victim & specific
transactions will only be necessary if the
Search warrant was issued by Municipal Court illegal transactions involved Specific/
judge, Judge Cancino to search and seize the office Isolated Transactions.
of FMS at Rizal Avenue for accounting records,
financial statements, books, records, ledgers, etc of
FMS.

Upon FMS filing writ of preliminary injunction and


petition for certiorari, CFI Judge, Judge Morfe
nullified the search warrant.

Morfe said that the search warrant was


unreasonable because:
If complainant really has personally knowledge of
these illegal activities happening in the FMS office
he should have known of specific banking
transactions/ victims.

“To authorize and seize all the records, w/o


reference to specific alleged victims of the
purported illegal banking transactions, would be to
harass the petitioner with a roving commission or
fishing expedition, for evidence which could be
discovered by normal intelligence operations or
inspection and not seizure of books and records.”

(this means that the search and seizure should not


be for ALL the records/ books/ financial statements
of the bank, but only those of the VICTIMS, or of
Specific Illegal Transactions)

Bache & Co. v. Ruiz Respondent, Judge Ruiz, issued a search warrant W/N the Search Warrant Search Warrant invalid. Ratio for requiring judge to personally
against Bache & Co. upon a letter from was Valid. examine complainant and witness:
Jill Commissioner of Internal Revenue, Vera. 1. Judge failed to PERSONALLY EXAMINE Art. 3, Sec. 3 requires that warrants be
W/N Corporations are the complainant and his witness. issued only upon probable cause. The
When the complainant (De Leon) and witness entitled to Protection determination of w/n a probable cause
(Logronio) came to the court, the Judge was against Unreasonable Revised Rules of Court Sec. 4 require that exists calls for the exercise of judgement
hearing another case at the sala. So, the judge Search and Siezure. the judge must personally examine on oath after a judicial appraisal of facts and should
instructed the Deputy Clerk of Court to take the or affirmation the complainant and his not be allowed to be delegated in the
depositions of De Leon and Logronio. witness and take their depositions in absence of an rule to the contrary.
writing, before he issues a Search Warrant.
After the Judge’s sessions, he was informed that the
depositions were already taken. So, Judge read When a Search Warrant may be said to
Judge basically just read the stenographer’s
Stenographer’s notes on the deposition and asked “PARTICULARLY DESCRIBE” the things
notes, gave a warning to the witness and
Logronio to take an oath. Judge also warned administered the oath. This cannot be seized:
Logronio that if he is lying he would be charged with considered personal examination or 1. The description is as specific as
perjury. Judge issued the search warrant. sufficient compliance with constitutional the circumstances will ordinarily
mandate. By reading the notes, Judge did allow
BIR agents were able to collect 6 boxes of files from not have the opportunity to observe the 2. When the description expresses a
the office of Bache. demeanor of witness and complainant, and conclusion of fact (not of law), by
to ask initial and follow-up questions, which the warrant officer may be
which the judicial mind, on account of its guided in making the search and
training, was in the best position to seizure.
conceive. 3. When the things described are
limited to those which bear direct
2. Search warrant was issued for more relation to the offense for which
than ONE specific offense. the warrant is being issued.

Revised Rules of Court Sec. 3 require that a Note: Stonehill v. Diokno was mentioned.
search warrant shall not issue but upon Respondent contend that the issue was not
probable cause in connection with one 1 specific offense. The offense in Diokno
specific offense… No search warrant shall was violation of “Central Banks Laws,
issue for more than 1 specific offense. Internal Revenue Code and RPC”. Court held
this case happened before the promulgation
The search warrant was issued for at least 4 of Revised Rules of Court. Thus, contention
not meritorious.
distinct offenses:
1. Non-compliance with proper
filling of income tax returns
2. Withholding of income taxes at
source
3. Unlawful pursuit of business or
occupation
4. Failure to make a return of
receipts, sales, etc, or failure to
pay the corresponding taxes

3. Search warrant does not PARTICULARLY


DESCRIBE the things to be Seized.
The term used: “Unregistered and private
books of accounts, receipts for payments
received….”
SC held that this list basically includes all
conceivable records of Bache, which is
seized could possibly render its business
inoperative.
It should have at least mentioned the dates,
amounts, persons or other pertinent data.

Corporations Entitled to protection against


unreasonable search and seizure
A corporation is but an association of
individuals under an assumed name with a
distinct legal entity. It organizing itself as a
collective body, it waives no constitutional
immunities appropriate to such body.

Soliven v. Makasiar Soliven broadcasted the statement that President 1. W/N petitioners were 1. NO. Issue is already moot and academic. What the constitution underscored is the
Aquino hid under her bed during a coup d' etat. The denied due process when Petitioner’s contention that they have been exclusive and personal responsibility of the
Katrine President sued for libel. Soliven claimed that he informations for libel were denied the administrative remedies issuing judge to satisfy himself of the
can't be sued because the President was immune filed against them although available under the law has lost factual existence of probable cause. In satisfying
from suit. the finding of the existence support. Due process of law does not himself of the existence of probable cause
of a prima facie cases was require that the respondent in a criminal for the issuance of a warrant of arrest, the
still under review by the case actually file his counter-affidavits judge is not required to personally examine
Secretary of Justice and before the preliminary investigation is the complainant and his witnesses.
subsequently, by the deemed completed. All that is required is Following the established doctrine and
President? that the respondent be given the procedure, he shall:
opportunity to submit counter-affidavits if
2. W/N the constitutional he is so minded. 1. personally evaluate the report and the
rights of Beltran were supporting documents submitted by the
violated when RTC judge 2. NO. It has not been shown that fiscal regarding the existence of probable
issued a warrant for his respondent Judge has deviated from the cause and, on the basis thereof, issue a
arrest without personally prescribed procedure. Thus, with regard to warrant of arrest; or
examining the complainant the issuance of the warrants of arrest, a
and the witnesses, if any, to finding of GAD amounting to lack or excess 2. if on the basis thereof he finds no
determine probable cause? or jurisdiction cannot be sustained. probable cause, he may disregard the
fiscal’s report and require the submission
3. W/N the president, under 3. YES. There is nothing in our laws that of supporting affidavits of witnesses to aid
the constitution, may would prevent the President from waiving him in arriving at a conclusion as to the
initiate criminal proceedings the privilege (immunity from suit). Thus, if existence of probable cause.
against the petitioners so minded, the President may shed the
through the filing of a protection afforded by the privilege and
complaint-affidavit? submit to the court’s jurisdiction. The
house of whether to exercise the privilege
or to waive it is solely the President’s
prerogative. It is a decision that cannot be
assumed and imposed by any other person.

Lim, Sr. v. Judge Felix On March 17, 1989, at about 7:30 o'clock in the WON a judge may issue a No. Respondent Judge committed a grave Soliven v. Makasiar: What the Constitution
morning, at the vicinity of the airport road of the warrant of arrest without error when he relied solely on the underscores is the exclusive and personal
Kats Masbate Domestic Airport, Congressman Moises bail by simply relying on the Prosecutor's certification and issued the responsibility of the issuing judge to satisfy
Espinosa, Sr. and his security escorts, Gaspar Amaro, prosecution's certification questioned Order dated July 5, 1990 himself of the existence of probable cause.
and Artemio Fuentes were attacked and killed by a and recommendation that a without having before him any other basis In satisfying himself of the existence of
lone assassin. Dante Siblante, another security probable cause exists. for his personal determination of the probable cause for the issuance of a
escort of Congressman Espinosa, Sr., survived the existence of a probable cause. warrant of arrest, the judge is not required
assassination plot but suffered a gunshot wound. to personally examine the complainant
In making the required personal and his witnesses. Following established
An investigation of the incident then followed. determination, a Judge is not precluded doctrine and procedures, he shall: (1)
from relying on the evidence earlier personally evaluate the report and the
The designated investigator at Camp Bagong Ibalon gathered by responsible officers. The extent supporting documents submitted by the
Legazpi City, Harry O. Tantiado, TSg, filed an of the reliance depends on the fiscal regarding the existence of probable
amended complaint with the Masbate MTC circumstances of each case and is subject to cause and, on the basis thereof, issue a
accusing, among others, Vicente Lim, Sr., Mayor the Judge's sound discretion. However, the warrant of arrest; or (2) if on the basis
Susana Lim, Jolly T. Fernandez, Florencio T. Judge abuses that discretion when having thereof he finds no probable cause, he may
Fernandez, Jr., Nonilon A. Bagalihog, Mayor Nestor no evidence before him, he issues a warrant disregard the fiscal's report and require the
C. Lim and Mayor Antonio Kho of the crime of of arrest. submission of supporting affidavits of
multiple murder and frustrated murder in witnesses to aid him in arriving at a
connection with the airport incident. conclusion as to the existence of probable
cause.
After conducting the preliminary investigation, the
court issued an order dated July 31, 1989 stating People v. Honorable Enrique B. Inting, et al.:
that a probable cause has been established for the emphasized important features of the
issuance of a warrant of arrest of named accused in constitutional mandate of Sec 2, Art 3: that
the amended complaint, namely, Jimmy Cabarles, ". . . no search warrant or warrant of arrest
Ronnie Fernandez, Nonilon Bagalihog, Jolly shall issue except upon probable cause to
Fernandez, Florencio Fernandez, Jr., Vicente Lim, Sr., be determined personally by the judge . . ."
Susana Lim, Nestor Lim, Antonio Kho, Jaime First, the determination of probable cause is
Liwanag, Zaldy Dumalag and Rene Tualla alias Tidoy. a function of the Judge. Second, the
preliminary inquiry made by a Prosecutor
The court ordered the arrest of the petitioners and does not bind the Judge. It merely assists
recommended the amount of P200,000.00 as bail him to make the determination of probable
for the provisional liberty of each of the accused. cause. And third, Judges and Prosecutors
On August 29, 1989, the entire records of the case alike should distinguish the preliminary
were transmitted to the Provincial Prosecutor of inquiry which determines probable cause
Masbate. for the issuance of a warrant of arrest from
the preliminary investigation proper which
On September 22, 1989, Fiscal Alfane issued a ascertains whether the offender should be
Resolution which affirmed the finding of a prima held for trial or released.
facie case against the petitioners but differed in the
designation of the crime. He ruled that the accused People v. Delgado, et al.: the court may
should be charged of MURDER for each of the killing require that the record of the preliminary
of the four victims and a physical injuries case for investigation be submitted to it to satisfy
inflicting gunshot wound on the buttocks of Dante itself that there is probable cause which will
Siblante. Vicente Lim, Sr. and Mayor Susana Lim warrant the issuance of a warrant of arrest.
filed a motion to reconsider the Resolution but such
motion was denied. Placer v. Villanueva: SC ruled that a judge
may rely upon the fiscal's certification of
On October 30, 1989, Fiscal Alfane filed with the the existence of probable cause and, on the
Masbate RTC, four (4) separate information of basis thereof, issue a warrant of arrest.
murder against the twelve (12) accused with a However, the certification does not bind the
recommendation of no bail. judge to come out with the warrant of
arrest.
On November 21, 1989, Vicente Lim, Sr. and Susana
Lim filed with the Supreme Court a verified petition
for change of venue.

On December 14, 1989, SC en banc issued a


Resolution authorizing the change of venue from
the Masbate RTC to Makati RTC to avoid a
miscarriage of justice. The cases were presided by
respondent, Judge Nemesio S. Felix.
Vicente Lim, Sr. and Susana Lim filed with the
respondent court several motions and
manifestations, which prayed for the following:
1. An order to be issued requiring the transmittal of
the initial records of the preliminary inquiry or
investigation conducted by the Municipal Judge
Barsaga of Masbate;
2. Movants to be given ample opportunity to file
their motion for preliminary investigation; and
3. In the event that this court may later be
convinced of the existence of a probable cause, to
be allowed to file a motion for reduction of bail or
for admission of bail.

In another manifestation, the Lims reiterated that


the court conduct a hearing to determine if there
really exists a prima facie case against them in the
light of documents which are recantations of some
witnesses in the preliminary investigation. The
motions and manifestations were opposed by the
prosecution.

On July 5, 1990, the respondent court issued an


order denying for lack of merit the motions and
manifestations and issued warrants of arrest against
the accused.

The petitioners then filed these consolidated


petitions questioning the July 5, 1990 Order.

In a Resolution dated July 17, 1990, SC issued a TRO,


ordering the respondent judge or his duly
authorized representatives or agents to cease and
desist from enforcing or implementing the warrant
of arrest without bail issued against the petitioners
in his Order dated July 5, 1990.

In another Resolution dated July 31, 1990, SC


resolved to issue writs of (1) PRELIMINARY
MANDATORY INJUNCTION, ordering and directing
the respondent judge to recall/set aside and/or
annul the legal effects of the warrants of arrest
without bail issued against and served upon the
petitioners; and (2) TEMPORARY RESTRAINING
ORDER, ordering the respondent judge or his duly
authorized representatives or agents, to CEASE AND
DESIST from enforcing or implementing the
warrants of arrest without bail issued against
petitioners, Mayors Nestor C. Lim and Antonio T.
Kho.

Webb v. De Leon NOTE: THIS IS THE CASE INVOLVING THE VIZCONDE 1. Whether or not the DOJ 1. NO. The DOJ Panel did not gravely abuse The need to find probable cause is dictated
MASSACRE Panel likewise gravely its discretion when it found probable cause by the Bill of Rights which protects "the
Jen abused its discretion in against the petitioners. A finding of right of the people to be secure in their
On June 19, 1994, the National Bureau of holding that there is probable cause needs only to rest on persons . . . against unreasonable searches
Investigation (NBI) filed with the Department of probable cause to charge evidence showing that more likely than not and seizures of whatever nature . . ." An
Justice a letter-complaint charging petitioners them with the crime of rape a crime has been committed and was arrest without a probable cause is an
and homicide committed by the suspects. Probable cause unreasonable seizure of a person, and
Hubert Webb, Michael Gatchalian, Antonio J. Lejano need not be based on clear and convincing violates the privacy of persons which
and six (6) other persons, 2 with the crime of Rape 2. Whether or not evidence of guilt, neither on evidence ought not to be intruded by the State.
with Homicide. Forthwith, the Department of respondent Judges de Leon establishing guilt beyond reasonable doubt Probable cause to warrant arrest is not an
Justice formed a panel of prosecutors headed by and Tolentino gravely and definitely, not on evidence establishing opaque concept in our jurisdiction.
Assistant Chief State Prosecutor Jovencio R. Zuño to abused their discretion absolute certainty of guilt. As well put in Continuing accretions of case law reiterate
conduct the preliminary investigation 3 of those when they failed to conduct Brinegar v. United States, while probable that they are facts and circumstances which
charged with the rape and killing on June 30, 1991 a preliminary examination cause demands more than "bare suspicion," would lead a reasonably discreet and
of Carmela N. Vizconde; 4 her mother Estrellita before issuing warrants of it requires "less than evidence which would prudent man to believe that an offense has
Nicolas-Vizconde, 5 and her sister Anne Marie arrest against them justify . . . conviction." A finding of probable been committed by the person sought to be
Jennifer 6 in their home at Number 80 W. Vinzons, cause merely binds over the suspect to arrested.
St., BF Homes, Parañaque, Metro Manila. 3. Whether or not the DOJ stand trial. It is not a pronouncement of
Panel denied them their guilt.
During the preliminary investigation, the NBI constitutional right to due
presented the following: process during their 2. NO. The Court ruled that respondent
preliminary investigation judges did not gravely abuse their
(1) the sworn statement dated May 22, 1995 of discretion. In arrest cases, there must be a
their principal witness, Maria Jessica M. Alfaro who 4. Whether or not the DOJ probable cause that a crime has been
allegedly saw the commission of the crime; Panel unlawfully intruded committed and that the person to be
(2) the sworn statements of two (2) of the former into judicial prerogative arrested committed it. Section 6 of Rule 112
housemaids of the Webb family in the persons of when it failed to charge simply provides that “upon filing of an
Nerissa E. Rosales and Mila S. Gaviola; Jessica Alfaro in the information, the Regional Trial Court may
(3) the sworn-statement of Carlos J. Cristobal who information as an accused. issue a warrant for the accused. Clearly the,
alleged that on March 9, 1991 he was a passenger our laws repudiate the submission of
of United Airlines Flight No. 808 bound for New York petitioners that respondent judges should
and who expressed doubt on whether petitioner have conducted “searching examination of
Webb was his co-passenger in the trip; witnesses” before issuing warrants of arrest
(4) the sworn statement of Lolita Birrer, a former against them.
live-in partner of Gerardo Biong, who narrated the What the Constitution underscores is the
manner of how Biong investigated and tried to exclusive and personal responsibility of the
cover up the crime at bar; issuing judge to satisfy himself of the
(5) the sworn statements of Belen Dometita and existence of probable cause. In satisfying
Teofilo Minoza, two of the Vizconde maids, and the himself of the existence of probable cause
sworn statements of Normal White, a security guard for the issuance of a warrant of arrest, the
and Manciano Gatmaitan, an engineer. The autopsy judge is not required to personally examine
reports of the victims were also submitted and they the complainant and his witnesses.
showed that Carmela had nine (9) stab wounds,
Estrellita twelve (12) and Jennifer nineteen (19). The 3. NO. The records will show that the DOJ
genital examination of Carmela confirmed the Panel did not conduct the preliminary
presence of spermatozoa. investigation with indecent haste.
Petitioners were given fair opportunity to
Petitioner Webb claimed during the preliminary prove lack of probable cause against them.
investigation that he did not commit the crime at Petitioners were afforded all the
bar as he went to the United States on March 1, opportunities to be heard. In addition to
1991 and returned to the Philippines on October 27, these, the panel even announced that any
1992. party may submit additional evidence
before the resolution of the case. The DOJ
On August 8, 1995, the DOJ Panel issued a 26-page Panel precisely allowed the parties to
Resolution "finding probable cause to hold adduce more evidence in their behalf and
respondents for trial" and recommending that an for the panel to study the evidence
Information for rape with homicide be filed against submitted more fully. This directly disputes
petitioners and their co-respondents. On the same the allegation of the petitioners that the
date, it filed the corresponding Information against resolution was done with indecent haste in
petitioners and their co-accused with the Regional violation of the rights of the petitioners.
Trial Court of Parañaque. The case was docketed as During the period of twenty-seven (27)
Criminal Case No. 95-404 and raffled to Branch 258 days, the petitioners were free to adduce
presided by respondent judge Zosimo V. Escano. It and present additional evidence before the
was, however, the respondent judge Raul de Leon, DOJ Panel.
pairing judge of Judge Escano, who issued the Verily, petitioners cannot now assert that
warrants of arrest against the petitioners. On they were denied due process during the
August 11, 1995, Judge Escano voluntarily inhibited conduct of the preliminary investigation
himself from the case to avoid any suspicion about simply because the DOJ Panel promulgated
his impartiality considering his employment with the adverse resolution and filed the
the NBI before his appointment to the bench. The Information in court against them.
case was re-raffled to Branch 274, presided by
Judge Amelita Tolentino who issued new warrants 4. NO. Petitioner's argument lacks appeal
of arrest against the petitioners and their for it lies on the faulty assumption that the
co-accused. On August 11, 1995, petitioner Webb decision whom to prosecute is a judicial
voluntarily surrendered to the police authorities at function, the sole prerogative of courts and
Camp Ricardo Papa Sr., in Bicutan, Taguig. beyond executive and legislative
Petitioners Gatchalian and Lejano likewise gave interference. In truth, the prosecution of
themselves up to the authorities after filing their crimes appertains to the executive
petitions before us. department of government whose principal
power and responsibility is to see that our
laws are faithfully executed. A necessary
component of this power to execute our
laws is the right to prosecute their violators.
The right to prosecute vests the prosecutor
with a wide range of discretion — the
discretion of whether, what and whom to
charge, the exercise of which depends on a
smorgasbord of factors which are best
appreciated by prosecutors. We thus hold
that it is not constitutionally impermissible
for Congress to enact R.A. No. 6981 vesting
in the Department of Justice the power to
determine who can qualify as a witness in
the program and who shall be granted
immunity from prosecution.

Ho v. People – 280 SCRA - On August 8, 1991, the Anti-Graft League of the May a judge issue a warrant NO, the judge may not. Soliven v Makasiar: the Consti underscores
365 Philippines, represented by Atty. Bagatsing, filed of arrest solely on the basis the exclusive and personal responsibility of
with the Ombudsman a complaint against Doris Ho of the report and - 1987 Consti requires judge to personally the issuing judge to satisfy himself of the
Robynne and Rolando Narciso (there are others but hindi sila recommendation of the determine probable cause existence of probable cause. In doing so,
part ng case) investigating prosecutor, judge is not required to personally examine
without personally - Respondent Sandiganbayan had only the the complaint.
- Complaint was for violation of RA No 3018 determining probable cause information filed by the Office of the
prohibiting a public officer from entering into any by independently examining Ombudsman when it issued the warrant of Following established doctrine and
contract//transaction on behalf of the government sufficient evidence arrest against the petitioners. The latter procedure, he shall:
if it is manifestly and grossly disadvantageous to the submitted by the parties two documents/reports even had dissimilar (1) personally evaluate the report and the
latter w/n the public officer profited or not. during the preliminary recommendations, his alone should have supporting documents submitted by the
investigation? prompted the public respondent to verify fiscal regarding the existence of probable
- Jan 8, 1992, Graft Investigation Officer Labrador the documents submitted by the parties cause and, on the basis thereof, issue a
(GIO Labrador) dismissed the case for insufficiency during the preliminary investigation, warrant of arrest
of evidence. whether there was sufficient evidence to (2) if on the basis thereof he finds no
sustain the Ombudsman’s action charging probable cause, he may disregard the fiscals
- After review of resolution of GIO Labrador, Special both petitioners with violation of Anti-Graft report and require the submission of
Prosecutor Officer Tamayo (SPO Tamayo) law. supporting affidavits of witnesses to aid him
recommended that Ho and Narciso be charge with in arriving at a conclusion as to the
violation of RA 3019 w/c was approved by - 3 vital matters (summary of existence of probable cause
Ombudsman Vasquez. jurisprudence) :
People v Inting: determination of probable
-The petitioner was chaged before the First, in Inting, the determination of cause entails:
Sandiganbayan probable cause by the prosecutor is for a
purpose different from that which is to be (1) judge alone determines probable cause,
(facts from information filed with Sandiganbayan) made by the judge. Whether there is (2) prelim inquiry of prosecutor doesnt bind
reasonable ground to believe that the the judge,
- On April 4, 1989, Narciso, VP of National Steel Corp accused is guilty is what the prosecutor (3) determination of probable cause for the
(NSC = GOCC) and Ho, president of National Marine passes upon. The judge, on the other warrant of arrest is made by the judge while
Corp (NMC = private corporation) conspired and hand, determines whether a warrant of the preliminary investigation proper (to
confederated with one to cause injury to NSC by arrest should be issued against the know if there is ground to believe accused is
entering w/o legal justification in a negotiated accused guilty) is the function of the prosecutor
contract of affreightment disadvantageous to NSC
for the haulage of its products from Iligan to Manila Second, the judge cannot rely solely on the People v Delgado: warrant issues not on
at Php 129.50 M/T. report of the prosecutor in finding the strength of the certification standing
probable cause to justify the issuance of a alone but because of the records which
- Petitioners knew that the rate was higher than warrant of arrest. Judge must decide sustain it.
that offered by Loadstar Shipping Company and independently. Must have evidence other
Premier Shipping Company at 109.56 and 123 M/T the prosecutor’s bare report. Allado v Diokno: Probable cause for the
respectively. This gave unwarranted benefits to issuance of a warrant of arrest is the
NMC of more than 1M to the prejudice and Lastly, What is required is that the judge existence of such facts and circumstances
pecuniary damage to NSC. must have sufficient supporting that would lead a reasonably discreet and
documents upon which to make his prudent person to believe that an offense
- Sandiganbayan issued warrant of arrest against independent judgment or, at the very has been committed by the person sought
petitioners. least, upon which to verify the findings of to be arrested. (KNOW THIS BY HEART
the prosecutor as to the existence of MAHIRAP NA hahaha)
-Petitioners alleged that Sandiganbayan, in probable cause. He cannot rely solely and
determining probable cause relied on the entirely on the prosecutors - The judge must personally evaluate the
recommendation.
information and resolution attached filed by the evidence in determining probable cause
Ombudsman w/o other supporting evidence, - Respondent Courts findings of “the
violated the requirements of Art 3, sec 2 of the conduct of a due and proper preliminary Webb v De Leon: judge merely determines
Constitution investigation” and “the approval by proper probability of guilt of the accused
officials clothed with statutory authority”
are not equivalent to the independent and
personal responsibility required by the
Constitution and settled jurisprudence.

People v. Francisco Verona and his live-in girlfriend, defendant W/N the search conducted THE SEARCH WAS INVALID. For the validity of a search warrant, the
Francisco, were placed under surveillance after the by the police officers at Constitution requires that there be a
Kat police confirmed through a test-buy operation that defendant’s residence was The application for search warrant filed by particular description of the place to be
they were engaged in selling shabu. Police officers valid Teneros requested for authority to search searched and the persons or things to be
Teneros and San Juan applied for a search warrant specifically the premises of No. 122M, seized.
before the RTC of Manila to search the premises of Hizon Street, Caloocan City. It was · A description of a place
respondents in 122 M. Hizon Street, Caloocan City. accompanied by a sketch of the area. to be searched is
Attached to the application was the During the hearing for the application of sufficient if the officer
After-Surveillance report of Teneros, which stated the warrant, Baradilla also described the with the warrant can,
that Baradilla, one of Verona’s runners in his drug house to be searched. When the trial court with reasonable effort,
operations, allegedly sought the assistance of then conducted an ocular inspection of the ascertain and identify
Teneros for Verona’s arrest. 122M Hizon Street, they found that it was the place intended and
two houses away from defendant’s house. distinguish it from other
Judge Bayhon issued a search warrant authorizing Meanwhile, 120 M Hizon Street was a places in the
the police officers to search for shabu and compound of 3 apartments, each of which community. Any
paraphernalia at No. 122M, Hizon Street, Caloocan were not numbered separately. designation or
City. description known to
In People v. Veloso, the SC held that: even a the locality that leads
Francisco, who was pregnant at the time, was inside description of the place to be searched is the officer unerringly to
the apartment when 8 policemen conducted the sufficient if the officer with the warrant it satisfies the
search. She inquired as to their identities but they can with reasonable effort, ascertain and constitutional
refused to answer and it was only at the police identify the place intended. requirement.
station that she found out that the policemen were ·
part of Teneros’ team. The team, along with This is different from the case at bar. From The requisites for the issuance of a valid
Barangay Chairwoman Limp and Kagawad de Jesus the application of the search warrant, as search warrant are:
of Caloocan, enforced the warrant and seized the well as the search warrant itself, the police 1. Probable cause is present
drugs and related paraphernalia they found, along officer serving the warrant cannot with 2. Such presence is determined
with cellular phones, television and monitoring reasonable effort ascertain and identify the personally by the judge
devices, P180,000, a Fiat car, jewelry, set of keys, an place intended precisely because it was 3. Complainant and the
ATM card, bank books and car documents. wrongly described as 122M, although it witnesses he or she may
Subsequently, Francisco was charged with violation may have been located in the same street produce are personally
of the Dangerous Drugs Act of 1972. as 120M. Even Baradilla’s description was examined by the judge, in
referring to the house at 122M, and not writing and under oath or
She filed a motion to quash the search warrant, 120M. affirmation
asserting that she and Verona were leasing the 4. Applicant and the witnesses
apartment at unit No. 120 M Hizon Street, testify on the facts
Caloocan City. RTC denied the motion to quash and The particularity of the place described is personally known to them
upheld validity of the search warrant. Defendant essential in the issuance of search warrants 5. Warrant specifically
appealed to the Supreme Court. to avoid the exercise by the enforcing describes the place to be
officer of discretion. Thus, the trial court searched and the things to
erred in refusing to nullify the actions of the be seized
police officers who were perhaps swayed by Absence of any of these requisites will
their knowledge of the place. The cause the downright nullification of the
controlling subject of search warrants is the search warrants.
place indicated in the warrant itself and not
the place identified by the police.

Thus, all the items seized during the illegal


search are prohibited from being used in
evidence.

HPS Software v. PLDT October 20, 2000, the complainant PAOCTF filed 2 W/N PLDT has the legal Court has categorically stated that ISR ISR is an alternative call pattern employed
applications of a search warrant for violation of personality to file the activity is an act of subtraction covered by by a communication provider outside the
Nat Article 308 of the RPC for Theft of Telephone petition. provisions on theftand the business of country. This is a method of routing and
Services and violation of P.D. 401 for unauthorized providing telecommunication or telephone completing international long distance call
installation of telephone communication W/N PLDT’s petition for service is a personal property (Strocheker v. using prepaid cards which respondents are
equipments following the complaint of PLDT that certiorari should have been Ramirez), which can be the object of theft selling. These calls are made through access
they were able to monitor the use of the dismissed because no under Art. 308 of RPC. calls that bypass the monitoring system,
respondents in their premises of Mabuhay cards motion for reconsideration #1 which makes the international long distance
and equipment capable of receiving and was filed by PLDT on the PLDT has the legal personality to file the calls appear as local calls, to the damage
transmitting calls from the USA and the Philippines May 23, 2001 Joint Order. petition. What is involved here is a search and prejudice of PLDT who is deprived of
without these calls passing through the facilities of warrant proceeding which is not a criminal revenue.
PLDT. W/N PLDT commited forum action much less a civil action, but a special
Complainant’s witnesses Richard Dira and Reuben shopping criminal process. Validity of a search warrant:
Hinagdan testified under oath that the respondents Since the search warrant is not a criminal - Must be issued on probably cause
are engaged in the business of International Simple W/N 2 search warrants action, it necessarily follow that the - Probable cause must be
Resale or unauthorized resale of international long were improperly quashed. requirement set forth in Section 5, Rule 110 determined by the judge himself
distance calls (ISR). W/N the search warrants of the Rules of Criminal Procedure which - Determination of probable cause,
*ISR defined on the side were general warrants state that “all criminal actions either the judge must examine, under
The witnesses testified that they found out that the commenced by complaint or by information oath or affirmation, the
respondent’s are engaged in the business of ISR on W/N the release of the shall be prosecuted under the direction and complainant and such witnesses
Sept. 13, 2000 when they conducted a test call items seized are proper. control of a public prosecutor” does not as the latter may produce
using Mabuhay Card. They testified that the test apply - Warrant issued must particularly
calls passing through the Mabuhay cards are being #2 describe the place to be searched
reflected as local calls only and not overseas calls. It is apparent that PLDT was deprived of and persons and things to be
Upon verification, they discovered that the lines due process when the trial court seized.
were subscribed by Philip YAp whose address is HPS expeditiously released the items seized by
Software and Communication Corp. (HPS) at Plaridel virtue of the subject search warrants
st. Alang-alang, Madaue City. These lines were without waiting for PLDT to file its A search warrant issued must particularly
transferred to HPS of the same address. They memorandum and despite the fact that no described the place to be searched and
further testified that the respondents committed motion for execution was filed by persons or things to be seized in order for it
these crimes by installing telecommunication respondents which is required in this case to be valid, otherwise, it is considered as a
equipments like multiplexers, lines, cables, because as stated the May 23, 2001 Joint general warrant which is proscribed by both
computers, and other switching equipment in the Order is a final order which disposes of the jurisprudence and the 1987 constitution.
HPS and connected these with PLDT telephone action or proceeding and which may be The purpose and intent of this requirement
lines. subject of an appeal. Therefore it is not is to limit the tings to be seized to those,
Satisfied with the affidavits and sworn testimonies immediately executory. and only those, particularly described in the
of the complainant’s witnesses the court issued The subject items were already released search warrant.
search warrants to seize the instruments of the without waiting for a motion for A search warrant may be said to particulary
crime. reconsideration, which would then be describe the things to be seized when the
useless. description therein is specific as the
October 20,2000 the trial court issued 2 search #3 circumstances will ordinarily allow; or when
warrants for the violation of Art. 308 of RPC and for PLDT was not engaged in forum shopping. the description expresses a conclusion of
the violation of P.D. 401. Both of which contained Both cases had different causes of action. fact - not of law - by which the warrant
identical orders directing that several items are to The appeal to the CA is an examination of officer may be guided in making the search
be seized from the premises of HPS. the validity of the trial court’s action of and seizure of when the things described
They were immediately implemented by a quashing the search warrants that it initially are limited to those which bear direct
PAOCTF-Visayas team. The police searched the HPS issued, while the petition for certiorari is an relation to the offense for which the
building and seized the articles in the search inquiry on w/n the trial court judge warrant is being issued.
warrants. committed grave abuse of discretion when
A preliminary investigation was conducted by the he ordered the release of the seized items.
Assistant City Prosecutor (Cotescon) of the Office of #4
the Prosecutor of Mandaue City. There was * see side for requirements of a valid search
probable cause found that all the crimes charged warrant
were committed and that Philip Yap, Hyman Yap, Trial Court and CA agree that no probable
Stanley Yap, Elaine Joy Yap, Julie Sy, Gene Bonial, cause existed to justify the issuance of the
Michael Pozon, John Doe and Jane Doe were warrants.
probably guilty thereof. The trial court anchored its determination
of probable cause for the issuance of the
November 23, 2000 Yap and HPS filed a motion to warrants on the sworn statements of the
quash the search warrants at issue on the ground PLDT personnel that the test calls had been
that the same did not refer to a specific offense; made using the subject Mabuhay Card.
there was no probable cause; and the search However, the facts were later proven to be
warrants were general warrants and wrongly incorrect.
implemented. PLDT opposed this by filing a CA concluded that contrary to PLDT’s claims
Consolidated Opposition. there were no test calls made because the
Trial court conducted hearings to W/N to quash the Mabuhay Card still had a balance of $10.
search warrants. Both sides presented their The SC on the other hand state that the
evidences. In the course of Engr. Tolentino’s search warrants are valid. The
testimony he identified certain pieces of evidence, determination of w/n test calls were made
which PLDT marked as its own exhibit, but was by PLDT cannot be ascertained solely by
objected by HPS on grounds of immateriality. The checking the value reflected on the
trial court sustained the objection and disallowed Mabuhay Card.
the production of the said exhibit so PLDT filed a There are several plausible reasons why the
manifestation with tender of excluded evidence. value of the Mabuhay Card did not change.
On april 19, 2001 PLDT formally offered in evidence PLDT provided the following probable
as part of Engr. Tolentino’s testimony. Then it files a cause:
motion for time to file memorandum asking the trial -The mabuhay cards were non-”smart”
court they be allowed to submit a memorandum in which takes a considerable amount of time
support of its opposition to the motion to quash the before the true value of the card is
search warrants filed by HPS. correspondingly reflected in the balacne.
However, the trial court issued the assailed Joint -PLDT suggested that the test calls that
Order on May 23, 2001, before the period for the were conducted in NTC-Region VII on
filing of PLDT’s memorandum had lapsed. November 3, 2000 and in open court
When PLDT discovered this development, it filed a January 10,2001 were long after the search
Notice of Appeal on june 7, 2001. warrant were issued (october 20, 2000).
PLDT likewise asserted that, without their The Mabuhay cards were already a matter
knowledge, the trial court caused the release to HPS of judicial record and thus easily
all of the seized items that were in the custody and ascertainable by either party. PLDT seerts
possession of PNP Task Force Group- Visayas. that this could have provided HPS an
opportunity to examine and identify the
July 18, 2001 PLDT filed a petition for certiorari specific mabuhay card and manipulate the
under rule 65 with the CA assailing the trial court’s remaining value in the card. In support of
released of the seized equipment despite the fact this tampering theory, PLDT points to the
that Joint Order dated May 23, 2001 had not yet HPS vehement opposition to introduce a
attained finality. new mabuhay card during the testimony of
CA granted PLDT’s motion and set aside the trial Engr. Tolentino.
court's May 23, 2001 Joint Order insofar as it Since there are other evidences submitted
released the seized equipments. by PLDT, SC finds that there are more than
HPS moved for reconsideration but was denied. sufficient evidence to engender a belief
that HPS had probably committed the crime
of theft through illegal IRS.
#5
* see side for notes on search warrants
The search warrants are not general
warrants because the items to be seized
were sufficiently identified physically and
were also specifically identified by stating
their relation to the offenses charged.
#6
Release of the seized items was enforced
prematurely and without any previous
motion for execution on record. The May
23, 2001 Joint Order of the trial court was
not immediately executory because the
period for appeal has not yet prescribe.

WWW Corp v. People Police Chief Inspector Napoleon Villegas of the I. W/N the CA erred in I. 1. An application for a search (The Court held them that they are liable
Regional Intelligence Special Operations Office giving due course to PLDT’s warrant is not a criminal action; for theft as all the elements are present –
RC (RISOO) of the PNP filed applications for warrants appeal despite the following confirmity of the public Just in case itanong kasi criminal law na
before the RTC of Quezon City to search the office procedural infirmities: prosecutor is not necessary to =)))
premises of Worldwide Web Corporation and Planet give the aggrieved party
Internet Corporation. The applications alleged that 1. PLDT, without the personality to question an order An application of search warrant is not a
petitioners were conducting illegal toll bypass conformity of the public quashing search warrants. criminal action
operations, which amounted to theft and violation prosecutor, had no
of P.D. No. 401 (Penalizing the Unauthorized personality to question the 2. An order quashing a search An application of search warrant is a judicial
Installation of Water, Electrical or Telephone quashal of the search warrant, which was issued process conducted either as an incident in a
Connections, the Use of Tampered Water or warrants. independently prior to the filing main criminal case already filed in court or
Electrical Meters and Other Acts), to the damage of a criminal action, partakes of a in anticipation of one yet to be filed.
and prejudice of the PLDT. 2. PLDT assailed the quashal final order that can be the proper
orders via an appeal rather subject of an appeal. Marshal case does not apply!
The trial court conducted a hearing on the than a petition for certiorari Quashal of the search warrants were final
applications for search warrants. The applicants under Rule 65 of the Rules II. Trial Judges determine probable orders, not interlocutory, and an appeal
Rivera and Gali of the Alternative Calling Pattern of Court cause in the exercise of their may be properly taken therefrom.
Detection Division of PLDT testified as witnesses. judicial functions. A trial judge’s
II. W/N the assialed search finding of probable cause for the Finding of probable cause for the issuance
Explanation on how WWW makes money tapping warrants were issues upon issuance of a search warrant is of a search warrant is made by a trial judge,
PLDT: probable cause, considering accorded respect by reviewing the finding is accorded respect by reviewing
From Rivera: that the acts complained of courts when the finding has courts. It is presumed that a judicial
So basically, WWW usues PLDT lines to connect with allegedly do not constitue substantial basis. It is a matter function has been regularly performed
international calls for free as they by pass its IGF – theft. wholly dependent on the finding absent a showing to the contrary.
toll by pass. This scheme constitutes toll bypass, a of trial judge in the process of
"method of routing and completing international III. Whether CA seriously exercising their judicial function. General Warrants - searh or arrest warrant
long distance calls using lines, cables, antenna erred in holding that the III. The requirement of particular in the that is not particular as to the person to be
and/or wave or frequency which connects directly assailed search warrants description of things to be seized arrested or the property to be seized.
to the local or domestic exchange facilities of the were not general warrants. is fulfilled when the items (VOID!!! It gives the officer executing the
originating country or the country where the call is described in the search warrant warrant discretion which items to take -
originated." bear a direct relation to the which is abhorrent - vurnerable to abuses)
offense for which the warrant is
From Gali: sought. Items which are technical in nature - Any
A PLDT phone number which is registred to WWC description of the place or thing or things to
was used to connect with a GlobalTalk. They be searched that will enable the officer
conducted a test call (Using PLDT line 6891135), the making the search with reasonable
given access line. After a voice prompt required him certainty to such place or thing is sufficient.
to enter the user code and PIN provided under a
GlobalTalk prepaid account, he was then requested
to enter the destination number, which included the Seized items:
country code, phone number and a pound sign. The 15 CPUs, 10 monitors, numerous wires,
call was completed to a phone number in Taiwan. cables, diskettes and files, and a laptop
computer. Petitioner contends that even
However, when he checked the records, it showed personal files and materials like diskettes
that the call was only directed to the local number were taken and said that they are not used
6891135. This indicated that the international test for transmission of interntaional calls.
call using GlobalTalk bypassed PLDT’s IGF. It also
found out that Planet Internet also uses the same Grounds for motion to quash : (1) the
number nad the same office address (so they are search warrants were issued without
one and the same). probable cause, since the acts complained
of did not constitute theft; (2) toll bypass,
Gali further alleged that because PLDT lines and the act complained of, was not a crime; (3)
equipment had been illegally connected by
petitioners to a piece of equipment that routed the the search warrants were general warrants;
international calls and bypassed PLDT’s IGF, they and (4) the objects seized pursuant thereto
violated P.D. No. 401 as amended, on unauthorized were "fruits of the poisonous tree." PLDT
installation of telephone connections. filed a Consolidated Opposition to the
motions to quash.
PLDT alleged that petitioners deprived it of foreign
exchange revenues, and evaded the payment of
taxes, license fees, and charges, to the prejudice of
the government.

During the hearing, the trial court required the


identification of the office premises/units to be
searched (NOTE! Office address of WWW is the
same with Planet Internet), as well as their floor
plans showing the location of particular computers
and servers that would be taken. The RTC granted
the application for search warrants. Three warrants
were issued against the office premises of
petitioners, authorizing police officers to seize
various items in the office premises of WWC and
Planet Internet, which includes various
telecommunications equipment consisting of
computers, lines, cables, antennas, modems, or
routers, multiplexers, PABX or switching equipment,
and support equipment such as software, diskettes,
tapes, manuals and other documentary records to
support the illegal toll bypass operations. The
warrants were implemented on the same day by
RISOO operatives of the NCR-PNP.

Over a hundred items were seized. (See the last


column for the list)

Planet Internet contends that Eastern and Capwire


both provided international direct dialing services,
which Planet Internet marketed by virtue of a
"Reseller Agreement." Planet Internet used PLDT
lines for the first phase of the call; but for the
second phase, it used the IGF of either Eastern or
Capwire. Planet Internet religiously paid PLDT for its
domestic phone bills and Eastern and Capwire for
its IGF usage. None of these contentions were
refuted by PLDT. So they say that they are not doing
anything that is prohibited.
MHP v. CA On February 22, 1983, petitioner MHP Garments, W/N there was valid seizure Article III, section 2:
No, evidence did not justify the warrantless
Inc., was awarded by the Boy Scouts of the
search and seizure of respondents’ goods:
Bryce Philippines, the exclusive franchise to sell and The right of the people to be secure in their
distribute official Boy Scouts paraphernalia. In their persons, houses, papers, and effects against
Memorandum Agreement, petitioner corporation 1. Progression of time between the receipt
unreasonable searches and seizures of
was given the authority to "undertake or cause to of the information and the raid of the
whatever nature for any purpose shall be
be undertaken the prosecution in court of all illegal stores shows there was sufficient time to
inviolable, and no search warrant or
sources of scout uniforms and other scouting apply for a judicial warrant.
warrant of arrest shall issue except upon
supplies." 2. No probable cause for the seizure
probable cause to be determined personally
Sometime in October 1983, petitioner received by the judge after examination under oath
information that private respondents Agnes Villa The members of the PC raiding team should or affirmation of the complainant and the
Cruz, Mirasol Lugatiman, and Gertrudes Gonzales have been included in the complaint for witnesses he may produce, and particularly
were selling Boy Scouts paraphernalia without any violation of the respondents’ constitutional describing the place to be searched and the
authority. Petitioner de Guzman, an employee of rights. Still, the omission will not exculpate persons or things to be seized.
the petitioner, was tasked to undertake the MHP Garments and De Guzman.
necessary surveillance and to make a report to the Rules of Court, a warrantless search can
Philippine Constabulary (PC). only be undertaken under the following
circumstance:
On October 25, 1983, at about 10:30 A.M.,
petitioner de Guzman, Captain Renato M. Peñafiel, Sec. 12. Search incident to a lawful arrest. -
and two (2) other constabulary men went to the A person lawfully arrested may be searched
stores of respondents at the Marikina Public for dangerous weapons or anything which
Market. Without any warrant, they seized the scout may be used as proof of the commission of
uniforms on display at respondents' stalls. The an offense, without a search warrant.
seizure caused a commotion and embarrassed
private respondents. Receipts were issued for the
seized items. The items were then turned over by
Captain Peñafiel to petitioner corporation for
safekeeping.

A criminal complaint for unfair competition was


then filed against private respondents. During its
pendency, petitioner de Guzman exacted from
private respondent Lugatiman the sum of THREE
THOUSAND ONE HUNDRED PESOS (P3,100.00) in
order to be dropped from the complaint. On
December 6, 1983, after a preliminary investigation,
the Provincial Fiscal of Rizal dismissed the complaint
against all the private respondents. On February 6,
1984, he also ordered the return of the seized
items. The seized items were not immediately
returned despite demands. Private respondents had
to go personally to petitioners' place of business to
recover their goods. Even then, not all the seized
items were returned. The other items returned
were of inferior quality. Private respondents then
filed Civil Case No. 51144 against the petitioners for
sums of money and damages.

In its Decision dated January 9, 1987, the trial court


ruled for the private respondents for return,
compensation, interest, damages and expenses.

The decision was appealed to the respondent court.


On January 18, 1989, its Fifth Division, affirmed the
Decision

In this petition for certiorari, petitioners contend:

First assignment of error


The court of appeals erred in imputing liability for
damages to the petitioners who did not effect the
seizure of the subject merchandise.

Second assignment of error


The court of appeals erred when it made a finding
that the manner with which the confiscation of
private respondents was tortuous but penalized
instead the petitioners who did not commit the act
of confiscation

Third assignment of error


The court of appeals erred when it found for the
private respondents and against the petitioners.

People v. CFI Regional Anti-Smuggling Action Center (“RASAC”) Regional Anti-Smuggling NO. The Court ruled that the circumstances PURPOSE OF THE CONSTITUTIONAL
was informed by an undisclosed Inforemer that a Action Center (“RASAC”) of the case at bar undoubtedly fall GUARANTEE AGAINST UNREASONABLE
Mar shipment of highly dutiable goods would be was informed by an squarely within the privileged area where SEARCHES AND SEIZURES
transported to Manila from Angeles City on a blue undisclosed Inforemer that search and seizure may lawfully be
Dodge car a week from then. a shipment of highly effected without the need of a warrant. - to prevent violations of private
dutiable goods would be security in person and property
On the day of the operation, at about 6:45a.m., the transported to Manila from Their decision was primarily based on the and unlawful invasion of the
RASAC agents gave a chase until they were able to Angeles City on a blue case of People v Mago, wherein the Court sanctity of the sanctity of the
corner and block the blue Dodge car driven by Sgt. Dodge car a week from in that case expressed the considered view home by officers of the law acting
Hope and accompanied by Medina. then. that “except in the case of a search of a under legislative or judicial
dwelling house, persons exercising police sanction and to give remedy
The Agents saw boxes on the back seat of the car On the day of the operation, authority under the customs law may effect against such usurpation when
and inquired as to its contents in which Sgt. Hope at about 6:45a.m., the search and seizure without warrant in the attempted.
replied that he doesn’t know. Agent Sabado RASAC agents gave a chase enforcement of customs laws.”
boarded the Dodge car and instructed Sgt. Hope on until they were able to GENERAL RULE: An intended search or
the route to take on their way to Tropical Hut corner and block the blue The Court further cited the traditional seizure attains a high degree of propriety
wherein Medina admitted was where they were Dodge car driven by Sgt. doctrine in the case of Caroll v United only when a probable cause duly
supposed to deliver the boxes. They made a brief Hope and accompanied by States stating that, “[s]earches and seizures determined is branded on a warrant duly
stop at Bonanza where Agent Manuel called up Col. Medina. without warrant are valid if made upon issued by a judge or other responsible
Abad to ask for instructions. probable cause, that is, upon a belief, person as may be authorized by law.
The Agents saw boxes on reasonably arising out of circumstances
The man who was supposed to pick up the boxes the back seat of the car and known to the seizing officer, than an EXCEPTION: In the extraordinary events
did not show up so Sgt. Hope, Medina and the inquired as to its contents in automobile or other vehicle contains that where warrant is not necessary to effect a
Dodge car were brought to Camp Aguinaldo and which Sgt. Hope replied that which by law is subject to seizure and valid search or seizure, or when the latter
arrived there at around 9a.m. An inspection of the he doesn’t know. Agent destruction.” cannot be performed except without
car yielded to 11 boxes containing watches and Sabado boarded the Dodge warrant, what constitutes a reasonable or
watch bracelets supposedly untaxed. As a car and instructed Sgt. Hope The Court concluded that the agents acted unreasonable search or seizure becomes
consequence thereof, they requested the Bureau of on the route to take on their not on the basis of a mere hearsay but on a purely a judicial question, determinable
Customs to issue a Warrant of Seizure and way to Tropical Hut wherein confirmed information worthy of belief and from the uniqueness of the circumstances
Detention against the articles and the car and such Medina admitted was probable cause enough for them to adopt involved.
was issued the same day. where they were supposed measures to freeze the fleeting event.
to deliver the boxes. They Therefore, what they did was a faithful
made a brief stop at performance of a duty authorized under NOTE: Dissenting and concurring opinion
Bonanza where Agent the Tariff and Customs Code. of Justice Teehankee
Manuel called up Col. Abad
to ask for instructions. He holds that the warrantless search and
seizure conducted by the RASAC agents in
The man who was supposed the case at bar should be invalidated and
to pick up the boxes did not the constitutional sanction declaring the
show up so Sgt. Hope, evidence obtained thereby “inadmissible
Medina and the Dodge car for any purpose in any proceeding” should
were brought to Camp be upheld.
Aguinaldo and arrived there
at around 9a.m. An He holds that the agents had enough time
inspection of the car yielded to acquire a valid warrant and whatever
to 11 boxes containing inconvenience which be caused by the
watches and watch delay in procuring such to Hope and
bracelets supposedly Medina would at least be tolerable
untaxed. As a consequence (quantifiable only in terms of hours while
thereof, they requested the waiting), rather than the transgression of
Bureau of Customs to issue their rights through the warrantless search
a Warrant of Seizure and and seizure which could be measured only
Detention against the in terms of fundamental constitutional
articles and the car and values violated.
such was issued the same
day. Furthermore, when the guarantee against
unreasonable search and seizure is invoked,
there is a need to scrutinize the facts
rigorously to preclude any infringement
thereof. He then cited the circumstances
around the cases used as basis by the
majority and how these exigent
circumstances justifying their proceeding to
do so without warrant were not present in
the instant case.

Roan v. Gonzales From the Case Book: Whether or not the search NO. The Court ruled that though there was EXAMPLE OF INSTANCES WHEN A SEARCH
warrant is valid thereby an application for the search warrant, it was MAY BE VALIDLY MADE WITHOUT
Mar PC Captain Mauro Quillosa personally filed his making the subsequent tainted with illegality by the failure of the WARRANT AND ARTICLES MAY BE TAKEN
application for a search warrant with two (2) search and seizure in Judge to conform with the essential VALIDLY AS A RESULT OF THAT SEARCH
witnesses, Esmael Morada and Jesus Tohilda, who accordance with the law. requisites of taking the depositions in
both presented the Judge with their respective writing and attaching them to record, (1) Warrantless search may be
affidavits. The judge also examined Captain Quillosa rendering the search warrant invalid. incidental to a lawful arrest, as
on the contents of the affidavit to ascertain, among Furthermore, the thing seized was beyond when the person being arrested is
others, if he knew and understood the same. The the scope of the search warrant issued, had frisked for weapons he may
judge also declared that he saw no need to have it been valid, for it does not follow that otherwise be able to use against
Quillosa’s deposition taken considering that he was because an offense is malum prohibitum, the arresting officer;
applying for a search warrant on the basis of the the subject thereof is necessarily illegal per (2) Motor cars may be inspected at
information provided by the witnesses whose se and may be summarily seized simply borders to prevent smuggling of
deposition had already taken by the judge. because they are prohibited as suggested aliens and contraband and even in
by the respondents. the interior upon a showing of
Part of the witnesses’ statements were: probable cause;
a) that they were suspicious of the The search was illegal because of the (3) Vessels and aircraft are also
petitioner because he was a follower of following reasons: traditionally removed from the
the opposition candidate in the a) the Judge did not ask his own operation of the rule because of
forthcoming election; searching questions but limited their mobility and their relative
b) that they saw eight (8) men deliver arms himself to the contents of the ease in fleeting the state’s
to the petitioner’s house and that Tohilda affidavit; jurisdiction;
said he saw everything through an open b) the applicant was asking for the (4) The individual may knowingly
window of the house while he was near issuance of a search warrant on agree to be searched or waive
the gate; and the basis of mere hearsay and not objections to an illegal search;
c) Tohilda can positively say that six (6) of of information personally known and
the weapons were .45 caliber pistols and to him; (5) Prohibited articles may be taken
twi (2) were .38 caliber revolvers. c) the Judge almost unquestioningly without warrant if they are open
received the witnesses’ to eye and hand and the peace
The search warrant was then issued as was used statements injudiciously omitted officer comes upon them
two (2) days later but none of the articles listed in judicious questions; and inadvertently.
the warrant was discovered. However, the officers d) petitioner did not voluntarily
conducting the search found in the premises one (1) submitted himself to the search REQUISITES IN ORDER OF A SEARCH
Colt Magnum Revolver and eighteen (18) live bullets for it was apparent that pressure WARRANT TO BE VALID
which they confiscated and used as bases of the was exerted by the military a) a search warrant must be
charges against the petitioner. authorities, who practically supported by probable cause to
coerced the petitioner to sign the be determined by the judge or
As added defense, the military officers also supposed waiver as a guaranty some other authorized officer
presented a waiver signed by the petitioner. against a possible challenge later after examining the complainant
to the validity of the search they and the witnesses he may
were conducting and the produce; and
petitioner had no choice but to b) there must be a specific
submit. description of the place to be
searched and the things to be
Though it may be true that there certain seized to prevent arbitrary and
instances when a search may be validly indiscriminate use of the warrant.
made without warrant and articles may be
taken validly as a result of that search but * It is axiomatic that the examination must
clearly though, the instant case does not be probing and exhaustive, not merely
come under any of the accepted routinary or pro-forma, if claimed
exceptions. Hence, the rule having been probable cause is to be established.
violated and no exception being applicable,
the conclusion is that the petitioner’s pistol * Mere affidavits of the complainant and
and bullets were confiscated illegally and his witnesses are not sufficient. The
therefore are protected by the exclusionary examining Judge has to take depositions in
principle. writing of the complainant and the
witnesses he may produce and attach
them to the record.

* While conceding that there may be


occasions when the criminal might be
allowed to go free because the constable
has blundered, the exclusionary rule is
nonetheless the only practical means of
enforcing the constitutional injunction
against abuse. It’s justification being that
only in case the prosecution which itself
controls the seizing officials, know that it
cannot profit by their wrong, will the
wrong be repressed.

People v. Malmstedt Malmstedt is a Swedish national touring in the W/N the arrest was lawfully Yes, Arreat valid. Probable Cause: facts and circumstances
Philippines. He went to Baguio then Sagada. On his made without a Search Arrest without warrant is lawful when, a which could lead a reasonable, discreet,
Jill way to back to Baguio. NARCOM (First Regional Warrant. Private person or, peace officer: and prudent man to believe that an offense
Command) received information that a bus coming 1. When, in his presence, the person to be has been committed, and that the objects
from Sagada will have a Caucasian carrying arrested has committed, is actually sought in connection to the offense are in
marijuana and other prohibited drugs. They committing, or is attempting to commit an the place sought to be searched.
received this information in the morning. offense.
2. When an offense has in fact just been Dissenting and Concurring opinion of
That afternoon, around 1:30pm, NARCOm had committed, and he has personal knowledge Narvasa:
already setup a checkpoint to check vehicles coming of facts indicating that the person to be Search INVALID. Officers did not have
from Cordillera region. The Bus Malmstedt rode was arrested has committed it personal knowledge that M actually
checked, and officers saw a Bulge in Malmstedt’s 3. When the person to be arrested is a committed a crime. M was also not actually
waist. When asked for his passport/ ID, he did not prisoner who has escaped from a penal committing a crime in the soldier’s
produce any. When asked to take out the bulge, it establishment or place where he is serving presence.
was found to be Hashish (a derivative of Marijuana). final judgment or temporarily confined Thus, all soldiers had was a mere suspicion
Upon opening his bags, they found teddy bears while his case is pending or has escaped that M might have some prohibited drugs.
filled with Hashish. while being transferred from one
confinement to another. There was an enumeration of several cases.

Accused was caught in flagrante delicto.


There was sufficient probable cause for the
officers to believe that the accused was
committing a crime.

There was also no time to obtain a search


warrant. NARCOM received the information
only a few hours before the bus actually
passed the checkpoint.

Posadas v. CA On October 16, 1986, Pat. Ursicio Ungab and Pat. W/N the warrantless search NO. Under Section 5, Rule 113 of the 1985 Section 5, Rule 113 of the 1985 Rules on
Umbra Umpar, both members of the INP of Davao is valid? Rules on Criminal Procedure, an arrest Criminal Procedure:
Katrine assigned with the Intelligence Task Force, were without a warrant may be effected by a
conducting a surveillance along Magallanes Street. peace officer or private person, among SEC. 5. Arrest without warrant; when
others, when in his presence the person to lawful — A peace officer or a private person
While they were within the premises of the Rizal be arrested has committed is actually may, without a warrant, arrest a person:
Memorial Colleges, they spotted petitioner carrying committing, or is attempting to commit an
a buri bag and they noticed him to be acting offence; or when an offence has in fact just (a) When in his presence, the person to be
suspiciously. They checked his bag and found 1 been committed, and he has personal arrested has committed is actually
caliber .38 Smith & Wesson revolver. 2 rounds of knowledge of the facts indicating that the committing, or is attempting to commit an
live ammunition for a .38 caliber gun, a smoke person arrested has committed it. However, offense
grenade and 2 live ammunitions for a .22 caliber the said circumstances in this case did not (b) When an offense has in fact just been
gun without a license. justify an arrest without a warrant. At the committed, and he has personal knowledge
time the peach officers identified of facts indicating that the person to be
Posadas was prosecuted for illegal possession of themselves and apprehended Posadas as arrested has committed it; and
firearms and ammunitions in the RTC which the CA he attempted to flee, they did not know
affirmed. that he had committed, or was actually (c) When the person to be arrested is a
committing the offence of illegal possession prisoner who has escaped from a penal
of firearms and ammunitions. They just establishment or place where he is serving
suspected that he was hiding something in final judgment or temporarily confined
the buri bag. They did not know what its while his case is pending, or has escaped
contents were. while being transferred from one
confinement to another.

In cases falling under paragraphs (a) and (b)


hereof, the person arrested without a
warrant shall be forthwith delivered to the
nearest police station or jail, and he shall be
proceeded against in accordance with Rule
112, Section 7. (6a, 17a)
Aniag v. COMELEC In preparation for the elections scheduled On 11 1. WON the warrantless 1. No. The warrantless search conducted by A warrantless search had been upheld in
May 1992, the COMELEC issued Resolution No. search conducted by the the Philippine National Police on 13 January cases of (1) moving vehicles (2) the seizure
Kats 2323 otherwise referred to as the "Gun Ban" on 11 PNP is legal. 1992 is illegal. of evidence in plain view and (3) search
December 1991. Subsequently, on 26 December conducted at police or military checkpoints
1991 COMELEC issued Resolution No. 2327 2. WON petitioner can be As a rule, a valid search must be authorized which are not illegal for as long as the
providing for the summary disqualification of validly prosecuted for by a search warrant duly issued by an vehicle is neither searched nor its
candidates engaged in gunrunning, using and instructing his driver to appropriate authority. However, this is not occupants subjected to a body search, and
transporting of firearms, organizing special strike return to the absolute. Aside from a search incident to a the inspection of the vehicle is merely
forces, and establishing spot checkpoints. Sergeant-at-Arms of the lawful arrest, a warrantless search had been limited to a visual search, and (4)
House of Representatives upheld in cases of moving vehicles and the Stop-and-search without warrant
On 10 January 1992, Mr. Serapio P. Taccad, the two firearms issued to seizure of evidence in plain view, as well as conducted by police officers on the basis of
Sergeant-at-Arms of the House of Representatives, him on the basis of the the search conducted at police or military prior confidential information which were
asked Aniag, the petitioner, to return the 2 firearms evidence gathered from the checkpoints which we declared are not reasonably corroborated by other attendant
issued to him by the House of Representatives. On warrantless search of his illegal per se, and stressed that the matters is also recognized by the court to
13 January 1992, petitioner immediately instructed car. warrantless search is not violative of the be legal.
his driver, Ernesto Arellano, to pick up the firearms Constitution for as long as the vehicle is
from petitioner's house at Valle Verde and return neither searched nor its occupants
them to Congress. subjected to a body search, and the
inspection of the vehicle is merely limited
At about five o'clock in the afternoon of the same to a visual search.
day, the PNP, headed by Senior Superintendent
Danilo Cordero, set up a checkpoint outside the An extensive search without warrant could
Batasan Complex some twenty (20) meters away only be resorted to if the officers
from its entrance. About thirty minutes later, the conducting the search had reasonable or
policemen manning the outpost flagged down the probable cause to believe before the search
car driven by Arellano. They searched the car and that either the motorist was a law offender
found the firearms neatly packed in their gun cases or that they would find the instrumentality
and placed in a bag in the trunk of the car. Arellano or evidence pertaining to the commission
was then apprehended and detained. He explained of a crime in the vehicle to be searched.
that he was ordered by petitioner to get the The existence of probable cause justifying
firearms from the house and return them to the warrantless search is determined by the
Sergeant-at-Arms Taccad of the House of facts of each case.
Representatives.
2. No.There was no sufficient evidence that
On 15 January 1992, the City Prosecutor ordered would impel the policemen to suspect
the release of Arellano after finding the latter's Arellano to justify the search they have
sworn explanation meritorious. conducted, such action constitutes an
unreasonable intrusion of the petitioner’s
On 28 January 1992, the City Prosecutor invited privacy and security of his property in
Aniag to shed light on the circumstances mentioned violation of Section 2, Article III of the
in Arellano's sworn explanation. Petitioner not only Constitution. Consequently, the firearms
appeared at the preliminary investigation to confirm obtained in violation of petitioner's right
Arellano's statement but also wrote the City against warrantless search cannot be
Prosecutor urging him to exonerate Arellano. He admitted for any purpose in any
explained that Arellano did not violate the firearms proceeding. The firearms seized during the
ban. warrantless search cannot be used as
evidence in any proceeding against
On 6 March 1992, the Office of the City Prosecutor petitioner.
issued a resolution which, among other matters,
recommended that the case against Arellano be
dismissed and that the "unofficial" charge against
petitioner be also dismissed.

Nevertheless, on 6 April 1992, COMELEC issued


Resolution No. 92-0829 directing the filing of
information against petitioner and Arellano for
violation of Sec. 261, par. (q), of B.P. Blg. 881
otherwise known as the Omnibus Election Code, in
relation to Sec. 32 of R.A. No. 7166; and petitioner
to show cause why he should not be disqualified
from running for an elective position, pursuant to
COMELEC Resolution No. 2327, in relation to Sec.
32, 33 and 35 of R.A. 7166, and Sec. 52, par. (c), of
B.P. Blg. 881.

On 13 April 1992, petitioner moved for


reconsideration but the COMELEC denied it.
Petitioner further maintains that he was neither
impleaded as party respondent in the preliminary
investigation before the Office of the City
Prosecutor nor included in the charge sheet.
Consequently, making him a respondent in the
criminal information would violate his constitutional
right to due process.

Malacat v. CA Rodolfo Yu of the Western Police District, Whether the search made NO. The search and was illegal. While The general rule as regards arrests, searches
Metropolitan Police Force of the Integrated National on Malacat is valid, probable cause is not required to conduct a and seizures is that a warrant is needed in
Jen Police, Police Station No. 3, Quiapo, Manila, pursuant to the exception of “stop and frisk,” it nevertheless holds that order to validly affect the same. The
testified that on 27 August 1990, at about 6:30 “stop and frisk.” mere suspicion or a hunch will not validate Constitutional prohibition against
p.m., in response to bomb threats reported seven a “stop and frisk.” A genuine reason must unreasonable arrests, searches and seizures
days earlier, he was on foot patrol with three other exist, in light of the police officer’s refers to those effected without a validly
police officers (all of them in uniform) along Quezon experience and surrounding conditions, to issued warrant, subject to certain
Boulevard, Quiapo, Manila, near the Mercury Drug warrant the belief that the person detained exceptions. As regards valid warrantless
store at Plaza Miranda. They chanced upon two has weapons concealed about him. Finally, arrests, these are found in Section 5, Rule
groups of Muslim-looking men, with each group, a “stop-and-frisk” serves a two-fold 113 of the Rules of Court. A warrantless
comprised of three to four men, posted at opposite interest: (1) the general interest of effective arrest under the circumstances
sides of the corner of Quezon Boulevard near the crime prevention and detection, which contemplated under Section 5(a) has been
Mercury Drug Store. These men were acting underlies the recognition that a police denominated as one “in flagrante delicto,”
suspiciously with their eyes moving very fast. officer may, under appropriate while that under Section 5(b) has been
circumstances and in an appropriate described as a “hot pursuit” arrest.
Yu and his companions positioned themselves at manner, approach a person for purposes of
strategic points and observed both groups for about investigating possible criminal behavior Turning to valid warrantless searches, they
thirty minutes. The police officers then approached even without probable cause; and (2) the are limited to the following:
one group of men, who then fled in different more pressing interest of safety and (1) customs searches;
directions. As the policemen gave chase, Yu caught self-preservation which permit the police (2) search of moving vehicles;
up with and apprehended petitioner. Upon officer to take steps to assure himself that (3) seizure of evidence in plain view;
searching petitioner, Yu found a fragmentation the person with whom he deals is not (4) consent searches;
grenade tucked inside petitioners front waist line. armed with a deadly weapon that could (5) a search incidental to a lawful arrest;
Yus companion, police officer Rogelio Malibiran, unexpectedly and fatally be used against and
apprehended Abdul Casan from whom a .38 caliber the police officer. (6) a “stop and frisk.”
revolver was recovered. Petitioner and Casan were
then brought to Police Station No. 3 where Yu Here, there are at least three (3) reasons
placed an X mark at the bottom of the grenade and why the “stop-and-frisk” was invalid: First,
thereafter gave it to his commander. there is grave doubts as to Yu’s claim that
Malacat was a member of the group which
The RTC ruled that the warrantless search and attempted to bomb Plaza Miranda 2 days
seizure of petitioner was akin to a stop and frisk. earlier. This claim is neither supported by
The RTC emphasized that Yu and his companions any police report or record nor
were confronted with an emergency, in which the corroborated by any other police officer
delay necessary to obtain a warrant, threatens the who allegedly chased that group. Second,
destruction of evidence and the officers had to act there was nothing in Malacat’s behavior or
in haste, as petitioner and his companions were conduct which could have reasonably
acting suspiciously, considering the time, place and elicited even mere suspicion other than
reported cases of bombing. The trial court thus that his eyes were “moving very fast” — an
found petitioner guilty of the crime of illegal observation which leaves us incredulous
possession of explosives under Section 3 of P.D. since Yu and his teammates were nowhere
No. 1866, and sentenced him to suffer reclusion near Malacat and it was already 6:30 p.m.,
temporal as minimum and reclusion perpetua as thus presumably dusk. Malacat and his
maximum. companions were merely standing at the
corner and were not creating any
CA-affirmed RTC’s decision but SC said that CA’s commotion or trouble. Third, there was at
decision should be set aside for having no all no ground, probable or otherwise, to
jurisdiction to hear this case. (Since the maximum believe that Malacat was armed with a
of the penalty is reclusion perpetua, the appeal deadly weapon. None was visible to Yu, for
therefrom should have been to SC, and not the as he admitted, the alleged grenade was
Court of Appeals) “discovered” “inside the front waistline” of
Malacat, and from all indications as to the
distance between Yu and Malacat, any
telltale bulge, assuming that Malacat was
indeed hiding a grenade, could not have
been visible to Yu.

People v. Aruta The accused-appellant Rosa Aruta was arrested and W/N the trial court erred in YES. There was really no excuse for the The right of a person to be secured against
charges with violating Section 4, Article 2, of RAnot finding that the NARCOM not to procure a search warrant any unreasonable seizure of his body and
Katrine 6425 or the Dangerous Drugs Act when she was warrantless search resulting considering that they had more than 24 any deprivation of his liberty is a most basic
caught in possession of marijuana. to the arrest of the hours to do so. While in principle we agree and fundamental one. A statute, rule or
accused-appellant violated that consent will validate an otherwise situation which allows exceptions to the
Her defence filed a Demurrer to Evidence alleging the latter constitutional illegal search, we believe that appellant -- requirement of a warrant of arrest or search
the illegality of the search and seizure of the items right? based on the transcript quoted above -- did warrant must perforce be strictly construed
violating her right against unreasonable search and not voluntarily consent to Bolonias search and their application limited only to cases
seizure as well as their inadmissibility in evidence. of his belongings. Appellants silence [when specifically provided or allowed by law. To
they searched her bag] should not be lightly do otherwise is an infringement upon
taken as consent to such search. The personal liberty and would set back a right
implied acquiscence to the search, if there so basic and deserving of full protection and
was any, could not have been more than vindication yet often violated.
mere passive conformity given under
intimidating or coercive circumstances and The following cases are specifically
is thus considered no consent at all within provided or allowed by law [to not have a
the purview of the constitutional warrant]:
guarantee. Furthermore, considering that 1. warrantless search incidental to a lawful
the search was conducted irregularly, i.e., arrest recognised under Section 12, Rule
without a warrant, we cannot appreciate 126 of the Rules of Court and by prevailing
consent based merely on the presumption jurisprudence
of regularity of the performance of duty 2. seizure of evidence in plain view, the
Obviously, this is again an instance of elements of which are:
seizure of the fruit of the poisonous tree, a. a prior valid intrusion based on
hence illegal and inadmissible subsequently the aid warrantless arrest in
in evidence. which the police are legally
present in the pursuit of their
official duties
b. the evidence was
inadvertently discovered by the
police who had the right to be
where they are
c. the evidence must be
immediately apparent
d. plain view justified mere
seizure of evidence without
further search
3. search of a moving vehicle - highly
regulated by the government, the vehicles
inherent mobility reduces expectation of
privacy especially when its transit in public
thoroughfares furnishes a highly
reasonable suspicion amounting to
probably cause that the occupant
committed a criminal activity
4. consented warrantless search
5. customs search
6. stop and frisk
7. exigent and emergency circumstances

The above exceptions, however, should not


become unbridled licenses for law
enforcement officers to trample upon the
constitutionally guaranteed and more
fundamental right of persons against
unreasonable search and seizures. The
essential requisite of probably cause must
still be satisfied before a warrantless search
and seizure can be lawfully conducted.

Asuncion v. CA On December 6, 1993, in compliance with the order Whether or not the NO. The search was legal the rule that Well-entrenched in this country is the rule
of the Malabon Municipal Mayor to intensify warrantless search was search and seizure must be supported by a that no arrest, search and seizure can be
Jen campaign against illegal drugs particularly at illegal and the shabu valid warrant is not absolute. The search of made without a valid warrant issued by
Barangay Taong, the Chief of the Malabon Police recovered, being illegally a moving vehicle is one of the doctrinally competent judicial authority. So sacred is
Anti-Narcotics Unit ordered his men to conduct obtained, was inadmissible accepted exceptions to the Constitutional this right that no less then the fundamental
patrol on the area with specific instruction to look as evidence. mandate that no search or seizure shall be law of the land ordains it.
for a certain vehicle with a certain plate number made except by virtue of a warrant issued However, the rule that search and seizure
and watch out for a certain drug pusher named Vic by a judge after personally determining the must be supported by a valid warrant is
Vargas. Pursuant thereto, SPO1 Advincula, PO3 existence of probable cause. The prevalent not absolute.
Parcon, PO3 Pilapil and a police aide were circumstances of the case undoubtedly bear
dispatched at around 11:45 in the evening. The out the fact that the search in question was The search of a moving vehicle is one of the
team proceeded to Barangay Taong where they made as regards a moving vehicle doctrinally accepted exceptions to the
were joined by their confidential informant and the petitioners vehicle was flagged down by the Constitutional mandate that no search or
latter informed them that a gray Nissan car is always apprehending officers upon identification. seizure shall be made except by virtue of a
parked therein for the purpose of selling shabu. Therefore, the police authorities were warrant issued by a judge after personally
While patrolling along Leoo Street, the confidential justified in searching the petitioners determining the existence of probable
informant pointed the gray Nissan car to the automobile without a warrant since the cause.
policemen and told them that the occupant thereof situation demanded immediate action. The
has shabu in his possession. The policemen apprehending officers even sought the
immediately flagged down the said car along First permission of petitioner to search the car,
Street and approached the driver, who turned out to which the latter agreed. As such, since
to be herein accused Jose Maria Asuncion y Marfori, the shabu was discovered by virtue of a
a movie actor using the screen name Vic Vargas and valid warrantless search and the petitioner
who is also known as Binggoy. Advincula then asked himself freely gave his consent to said
the accused if they can inspect the vehicle. As the search, the prohibited drugs found as a
accused acceded thereto, Advincula conducted a result were admissible in evidence.
search on the vehicle and he found a plastic packet
containing white substance suspected to be
methamphetamine hydrochloride/shabu (Exhibit
D-1) beneath the drivers seat. The accused told the
policemen that he just borrowed the said car and he
is not the owner thereof. The accused was
thereafter taken at the police headquarters for the
purpose of taking his identification. However, when
he was frisked by Advincula at the headquarters,
the latter groped something protruding from his
underwear, which when voluntarily taken out by the
accused turned out to be a plastic packet containing
white substance suspected to be
methamphetamine hydrochloride/shabu (Exhibit D).
A press conference was conducted the following day
presided by Northern Police District Director Pureza
during which the accused admitted that the
methamphetamine hydrochloride were for his
personal use in his shooting.

Asuncion claimed that the warrantless search was


illegal and that the shabu recovered, being illegally
obtained, was inadmissible as evidence.

People v. Canton - Respondent Susan Canton was charged W/N the warrantless NO!! Search was made pursuant to Exceptions established by jurisprudence
before RTC-Pasay with violation of sec 16, Art. search and subsequent routine airport security procedure, on warrantless search are:
Robynne III of Dangerous Drugs Act of 1972(RA No. 6425) seizure of the regulated which is allowed under Section 9 of (1) search of moving vehicles;
drugs, as well as the Republic Act No. 6235. (2) seizure in plain view;
- On Feb 12, 1998, Susan was is NAIA and was arrest of Susan were (3) customs searches;
bound for Vietnam. The metal detector beeped violative of her - What constitutes a reasonable or (4) waiver or consented searches;
prompting Mylene Cabunoc, civilian employee constitutional rights. unreasonable search in any particular (5) stop and frisk situations (Terry
of National Action Committee on Hijacking and case is a judicial question, determinable search); and
Terrorism and frisker on duty, called her from a consideration of the (6) search incidental to a lawful arrest
attention. Upon pat search, Mylene felt a bulge circumstances involved
at Susan’s abdomen area and took out the Section 1 of Rule 113 of the Revised
package which contained what felt like rice -Susan’s arrest didnt precede the Rules of Criminal Procedure: arrest is
granules. Same bulges were felt around her search. Strip search was for knowing the taking of a person into custody in
genital area and thigh. what the packages were. If there was order that he may be bound to answer for
deprivation of liberty, it didnt amount to the commission of an offense.
- Mylene called Customs Examiner Lorna Jalac an arrest. Prior to strip search in the
and brought Susan to restroom for a thorought restroom, Mylene had no knowledge of Terry search or the stop and frisk
examination. They found 3 individually wrapped what was hidden on Susan’s body, situation: a case where a police officer
packages which Susan voluntarily handed over meaning they didnt know yet whether approaches a person who is acting
to them. Mylene turned over packages to SP04 was crime was committed. It was only suspiciously, for purposes of
Victorio de los Reyes, her supervisor on duty, after discovery that she was arrested. investigating possibly criminal behavior
who informed his superior officer Police The search cannot, therefore, be said to in line with the general interest of
Superintendent Daniel Santos about the have been done incidental to a lawful effective crime prevention and detection.
incident. arrest. In a search incidental to a lawful
arrest, the law requires that there be first RA 6235 SEC. 9:
- The packages tested positive for a lawful arrest before a search can be
methamphetamine hydrochloride or shabu. made Every ticket issued to a passenger by the
- For the defense, SP02 Jerome Cause, airline or air carrier concerned shall
investigtor of First Regional Aviation Office, - In this case, search was made pursuant contain among others the following
testified no investigation was conducted on to routine airport security procedure condition printed thereon: Holder hereof
Susan but she signed a receipt of the article allowed under RA 6235.. and his hand-carried luggage(s) are subject
seized from her (packages, ticket). Susan also to search for , and seizure of, prohibited
had no counsel with her when she signed the - R.A. No. 6235 authorizes search for materials or substances. Holder refusing to
receipt. prohibited materials or substances. To be searched shall not be allowed to board
limit the action of the airport security the aircraft, which shall constitute a part of
- Trial court found Susan guilty beyond personnel to simply refusing her entry the contract between the passenger and
reasonable doubt. Susan filed for MR/New trial into the aircraft and sending her home the air carrier
and to inhibit Judge Porfirio Macaraeg for and depriving them of the ability act
partiality. accordingly, including to further search Rule 113 of the Rules of Court,
without warrant, would be to sanction provides:
- Trial court denied motions. Conviction was not ineffectivity in law enforcement, to the
based on medical report, no violation of Susan’s detriment of society. The strip search in
SEC. 5. Arrest without warrant; when
Consti rights because she was never the ladies room was justified under the
lawful. -- A peace officer or a private
interrogated during her detention w/o counsel, circumstances.
person may, without a warrant, arrest a
packages were seized during a routine frisk at person:
the airport. “Persons may lose the protection of
the search and seizure clause by
(a) When, in his presence, the person to
- Unsatisfied with decision, Susan appealed to exposure of their persons or property
be arrested has committed, is actually
SC imputing the following errors: (1) the to the public in a manner reflecting a
committing, or is attempting to commit an
warrantless search was based on probable lack of subjective expectation of
offense
cause; (2) in holding her flagrante delicto and privacy, which expectation society is
warrantless search was incidental to a lawful prepared to recognize as reasonable.
Such recognition is implicit in airport Custodial investigation: the
arrest; not ruling frisker when beyond limits of
security procedures” questioning initiated by law enforcement
the Terry search doctrine; (4) in not ruling that
- Ordinary Consti protections against officers after a person has been taken
SUSAN was under custodial investigation
warrantless arrest dont apply in routine into custody or otherwise deprived of his
without counsel; (5) in admitting the report of Dr.
airport procedures. freedom of action in any significant way
Ma. Bernadette Arcena, which was not testified
on or offered in evidence, and using the same in
determining her guilt; (6) in justifying under the - Search conducted resulted in the
rule on judicial notice its cognizance of the discovery of the packages which means
medical report that has not been offered in that Susan was committing a crime and
evidence; and (7) in applying the ruling in People the airport security were duty bound to
v. Johnson. arrest her.

- OSG: Susan was found flagrante delicto, case - No custodial investigation was
falls under exception being a warrantless search conducted after Susan’s arrest. No
incidental to lawful arrest, and she voluntarily statement was taken from her during her
submitted to the frisking. Susan’s conviction was detention and used in evidence against
also not based on med cert issued but on facts of her.
the case.

Fajardo v. People n Members of the Provincial Intelligence Special W/N the discovery of the 2 The receivers were seized in plain view and Our Constitution recognizes the right of
Operations Group (PISOG) were instructed by receivers comes within the hence admissible people to be secure in their persons,
Kat Provincial Director Superintendent Mendoza to purview of the PLAIN VIEW houses, papers and effects against
respond to a complaint by concerned citizens in DOCTRINE Under the plain view doctrine, objects unreasonable searches and seizures, as
Aklan that armed men drinking liquor at the falling in the “plain view” of an officer, who encapsulated in Article III, Section 2 of the
residence of petitioner were indiscriminately firing has a right to be in the position to have that Constitution.
guns. They proceeded to the area and upon their view, are subject to seizure and may be
arrival, they noticed that several persons scampered presented as evidence. It applies when the There are, however, several exceptions to
and ran in different directions. They saw Valerio, following requisites: this rule. Evidence obtained through a
one of the accused, holding 2 .45 caliber pistols. He warrantless search and seizure may be
fired at the policemen before entering petitioner, 1. Law enforcement officer in search of the admissible under any of the following
Fajardo’s house. Likewise, petitioner was seen evidence has a prior justification for an circumstances:
tucking a .45 caliber gun between her waist before intrusion or is in a position from which he 1. A warrantless search
entering her house. can view a particular area incidental to a lawful arrest
2. Search of a moving motor
To prevent commotion, the policemen desisted from (Since petitioner and Valerio were holding vehicle
entering the house but cordoned the perimeter of the pistols before they ran inside the house, 3. Search in violation of custom
the area and thereafter, awaited further the presence of Police Officer Nava at the laws
instructions from Mendoza. At around 2:00AM, back of the house was justified) 4. Seizure of evidence in plain
Nava, one of the police officers, saw Valerio emerge view
twice on the top of the house and throw something. 2. The discovery of evidence in plain view is 5. When the accused himself
The objects landed near the wall of petitioner’s inadvertent waives his right against
house and inside the compound of the neighboring unreasonable searches and
residence. Nava, together with 2 other policemen, (From where he was, Nava clearly saw seizures
recovered the discarded objects – which turned out Valerio emerge from the top of the house
to be 2 receivers of .45 caliber pistols. These were and throw the suspicious objects)
surrendered to police officer Tan, group
investigator, who utilized them in applying for and 3. It is immediately apparent to the officer
obtaining a search warrant. that the item he observes may be evidence
of a crime, contraband, or otherwise
Once the warrant was served on petitioner at subject to seizure
9:30AM, the police team proceeded to search the
petitioner’s house. The team found and was able to (Nava had reasonable ground to believe
confiscate more illegal firearms and ammunition. that the things thrown were contraband
Since petitioner and Valerio were unable to present items or evidence of the offense they were
any documents showing their authority to possess suspected of committing)
the confiscated items, they were charged for
violation of PD 1866. Tested against these standards, the seizure
of the 2 receivers of .45 caliber pistols
They argue that the issuance of the search warrant outside the petitioner’s house falls within
was defective because of the following: the purview of the plain view doctrine. The
2 receivers were admissible as evidence but
(1) 1. The allegation contained in the application filed the liability for their possession should fall
and signed by Tan was not based on his personal only on Valerio and not petitioner.
knowledge, as it was “founded on confidential
information received by the Provincial Director (Note: Only Valerio was convicted with
Police Superintendent Mendoza.” illegal possession of firearms for failure of
(2) 2. Execution of the search warrant was infirm the prosecution to support Fajardo’s
because petitioner was not asked to accompany the conviction with enough evidence to
policemen as they explored the house but was constitute proof beyond reasonable doubt.)
ordered to remain in the living room

RTC RULING
Petitioner and Valerio were convicted by RTC of
illegal possession of firearms. RTC also ruled that
petitioner was estopped from assailing the legality
of their arrest since they participated in the trial by
presenting evidence. Likewise applying for bail, they
effectively waived such irregularities and defects.

CA RULING
CA affirmed factual findings of RTC but held that the
search warrant was void since Tan did not have
personal knowledge of the fact that accused had no
license to possess firearms and that he failed to
attach a certification from Firearms and Explosives
Office of the PNP attesting such absence of license.
Resultantly, CA held that all firearms and explosives
found inside petitioner’s residence were
inadmissible as evidence but the 2 firearms
recovered by the policemen outside the house prior
to the issuance of the warrant were admissible as
evidence pursuant to the plain view doctrine.

Petitioner moved for reconsideration and when it was


denied, he filed the present petition with SC.

Luz v. People Police officer Alteza, a traffic enforcer assigned at 1. W/N the petitioner was 1. There was NO valid arrest of petitioner. The following are instances where a
Naga City, saw accused driving a motorcycle without validly arrested warrantless search is allowed:
Kat a helmet. This prompted him to flag down the When he was flagged down for committing 1. A warrantless search
accused for violating a municipal ordinance which 2. W/N the warrantless a traffic violation, petitioner was not, ipso incidental to a lawful arrest
requires all motorcycle drivers to wear helmets search conducted by Alteza facto and solely for this reason, arrested. 2. Search of evidence in “plain
while driving. Since the place he flagged down the was legal Arrest is the taking of a person into custody view”
accused was almost in front of their station, he in order that he or she may be bound to 3. Search of a moving vehicle
invited the accused inside. answer for the commission of an offense. It 4. Consented* warrantless
is effected by an actual restraint of the search
While he and another officer, Brillante, were issuing person to be arrested or by that person’s 5. Customs search
a ticket for the violation, he noticed that the voluntary submission to the custody to the 6. A “stop and frisk” search
accused was uneasy and kept getting something one making the arrest. 7. Exigent and emergency
from his jacket. Thinking that the accused may be circumstances
carrying a weapon, he ordered the accused to Under the Land Transportation and Traffic
empty the contents of his pocket, which contained Code, the procedure for dealing with a *Whether consent to the search was
therein a nickel-like tin container, 2 cellphones, 1 traffic violation is not the arrest of the voluntary is a question of fact to be
pair of scissors, and a swiss knife. When he ordered offender but the confiscation of the driver’s determined from the totality of all the
the accused to open the container, he saw that it license of the latter. In this case, according circumstances. Relevant to this
contained 2 plastic sachets of shabu. to the City Ordinance violated by petitioner, determination are the following:
his failure to wear a helmet is penalized by 1. Age of defendant
Upon arraignment, petitioner pleaded not guilty. a fine only. 2. Whether defendant was in a
During the trial, Alteza testified for the prosecution public or secluded location
while accused testified for himself and raised the At the time he was waiting for Alteza to 3. Whether defendant objected
defense of (1) planting of evidence and (2) write his citation ticket, petitioner could not to the search or passively
extortion. In spite of this, RTC convicted petitioner be said to have been “under arrest”. There looked on
of illegal possession of dangerous drugs. Upon was no intention on the part of Alteza to 4. Education and intelligence of
review, the CA affirmed RTC’s decision. Thus, arrest him, deprive him of his liberty or defendant
petitioner filed certiorari with the Supreme Court take him into custody. Prior to the issuance 5. Presence of coercive police
and claims that there was no lawful search and of his ticket, the period during which procedures
seizure because there was no lawful arrest. petitioner was at the police station was 6. Defendant’s belief that no
merely “waiting time”. It was only for the incriminating evidence
sake of convenience that they were there. would be found
7. Nature of the police
Even assuming that the petitioner was questioning
deemed arrested upon being flagged down 8. Environment in which the
for a traffic violation, the requirements for questioning took place
a valid arrest were not complied with. At 9. Possibly vulnerable
the time a person is arrested, it is the duty subjective state of the
of the officer to inform the latter of the person consenting
reason for his arrest and must show that
person the warrant of arrest, if any. Persons
shall also be informed of their
constitutional rights to remain silent and to
counsel, and that any statement they might
make could be used against them. In this
case, these requirements were complied
with by the officers only after petitioner
had been arrested for illegal possession of
dangerous drugs.

2. There being no valid arrest, the


warrantless search that resulted from it
was likewise illegal

In addition to the invalid arrest:


1. The evidence seized in this case
was not in “plain view” and was
actually concealed in a metal
container inside petitioner’s
pocket.

2. No consent was given by


petitioner to the warrantless
search. The fact that petitioner
acceded to Alteza’s instruction to
empty his pockets does not prove
consent.

Disini v. Sec. of Justice As far as Article 3 Sec 2 is concerned: W/N Sec. 14, 15 and 19 are Sec 14
valid and constitutional on Yes, what Section 14 envisions is merely the
Bryce the grounds of Article 3 Sec.
Sec. 14. Disclosure of Computer Data. — Law 2. enforcement of a duly issued court warrant,
enforcement authorities, upon securing a court a function usually lodged in the hands of
warrant, shall issue an order requiring any person or law enforcers to enable them to carry out
service provider to disclose or submit subscriber’s their executive functions. The prescribed
information, traffic data or relevant data in his/its procedure for disclosure would not
possession or control within seventy-two (72) hours constitute an unlawful search or seizure nor
from receipt of the order in relation to a valid would it violate the privacy of
complaint officially docketed and assigned for communications and correspondence.
investigation and the disclosure is necessary and Disclosure can be made only after judicial
relevant for the purpose of investigation. intervention.

Sec. 15. Search, Seizure and Examination of Sec 15


Computer Data. — Where a search and seizure Yes, Section 15 merely enumerates the
warrant is properly issued, the law enforcement duties of law enforcement authorities that
authorities shall likewise have the following powers would ensure the proper collection,
and duties. preservation, and use of computer system
Within the time period specified in the warrant, to or data that have been seized by virtue of a
conduct interception, as defined in this Act, and: court warrant. The exercise of these duties
do not pose any threat on the rights of the
(a) To secure a computer system or a computer data person from whom they were taken.
storage medium; Section 15 does not appear to supersede
(b) To make and retain a copy of those computer existing search and seizure rules but merely
data secured; supplements them.
(c) To maintain the integrity of the relevant stored
computer data; Sec. 19
(d) To conduct forensic analysis or examination of No, Section 2, Article III of the 1987
the computer data storage medium; and Constitution provides that the right to be
(e) To render inaccessible or remove those secure in one’s papers and effects against
unreasonable searches and seizures of
computer data in the accessed computer or whatever nature and for any purpose shall
computer and communications network. be inviolable. Further, it states that no
search warrant shall issue except upon
Pursuant thereof, the law enforcement authorities probable cause to be determined
may order any person who has knowledge about personally by the judge. Here, the
Government, in effect, seizes and places the
the functioning of the computer system and the
computer data under its control and
measures to protect and preserve the computer disposition without a warrant. The
data therein to provide, as is reasonable, the Department of Justice order cannot
necessary information, to enable the undertaking of substitute for judicial search warrant.
the search, seizure and examination.
Law enforcement authorities may request for an
extension of time to complete the examination of
the computer data storage medium and to make a
return thereon but in no case for a period longer
than thirty (30) days from date of approval by the
court.

Sec. 19. Restricting or Blocking Access to Computer


Data.— When a computer data is prima facie found
to be in violation of the provisions of this Act, the
DOJ shall issue an order to restrict or block access to
such computer data.

Petitioners’ objection to Section 14 is that the


issuance of subpoenas is a judicial function. But it is
well-settled that the power to issue subpoenas is
not exclusively a judicial function. Executive
agencies have the power to issue subpoena as an
adjunct of their investigatory powers.

Petitioners challenge Section 15 on the assumption


that it will supplant established search and seizure
procedures.

Petitioners contest Section 19 in that it stifles


freedom of expression and violates the right against
unreasonable searches and seizures

Material Distributors v. Case Book (These facts are from the original since W/N the trial court judge The reasons advanced by the petitioners
Natividad it’s hard to understand the case book) violated petitioner’s from their pleadings were carefully
constitutional right against considered and nothing was found to show
Nat March 24, 1947, Lope Sarreal filed a complaint self-incrimination how, without the inspections of Annexes A
(amended on April 10, 1947, to include Harry Lyons) and B of the complaint, petitioners may
seeking a money judgment against petitioners on W/N the trial court judge incriminate themselves. So their contention
three causes of action in the total of P1,256,229.30. violated Section 1 article 3 is dismissed.
On May 27, 1947, Sarreal filed a motion for the of the constitution on illegal
production and inspection of certain documents search and seizure. The order of the trial judge, issued in virtue
(Books or Papers of Material Distributors Inc., and of the provision of Rule 21, pertain to a civil
Books and Papers of defendant Harry Lyons. W/N the trial court judge procedure that cannot be identified or
On June 4, 1947, Sarreal filed a supplemental violated the constitutional confused with the unreasonable searches
motion for the production and inspection of the guarantee of privacy of prohibited by the Constitution. The
originals of Annexes A and B of the complaint. erroneous hypothesis that the production
(Application of Section 1 Rule 21) communication and the inspection of books and documents
in question is tantamount to a search
On July 16, 1947, respondent judge required the warrant, the procedure outlined by Rule 21
petitioners to produce the documents and annexes and followed by the respondent judge place
in question on July 24, 1947. them outside the realm of prohibited
unreasonable searches.
Petitioners contend that in filing his original and Sarreal has an interest in the books and
supplemental motions, Sarreal has failed to show documents in question and they are
good cause for the issuance of the requested order. material and important to the issue
It appears, however, in the original motions of May between him and the petitioners. Justice
27, that the books and papers mentioned will be better served if all the facts
“constitute or contain the evidence material to the pertinent to the controversy are placed
matters involved in the above entitled case.” before the trial court.

In the supplemental motion of June 4, 1947, it is The constitutional guarantee of privacy of


alleged that there is direct conflict between the communication and correspondence will
allegations of the complaint and amended not be violated, because the trial court has
complaint and those of the answer and amended power and jurisdiction to issue the order for
answer as to whether or not the names of Gil J. the production and inspection of the books
Puyat and Raymond W. Lehmann appear in any part and documents in question in virtue of the
of the originals of Annexes A and B of the constitutional guarantee making an express
complaint, and plaintiff Sarreal wanted the exception in favor of the disclosure
production and inspection of said originals to show communication and correspondence upon
that they did not contain the names of Gil J. Puyat lawful order of a court of justice.
and Raymond W. Lehmann, and that if said names
should appear now typed in said Annexes A and B, The trial judge, in issuing the order of July
said additional names must have been typed by 16, 1947, has not exceeded his jurisdiction
direction of Harry Lysons without the knowledge or or acted with grave abuse of discretion.
consent of Sarreal and after said originals were
delivered by Harry Lysons and filed by the latter and
that the changes so introduced are a forgery.

With these allegations in the original and


supplemental motions Sarreal has fulfilled the
requirements of showing good cause for the
production and inspection of the books and
documents in question under Rule 21.

Oklahoma Press v. Walling Non-judicial subpoenas issued by an WON non-judicial Yes. The very purpose of the subpoena Dissenting Opinion (Justice Murphy):
administrative agency sought the production of subpoenas issued by an and of the order is to discover and
RC specified records in order to determine whether administrative agency is procure evidence, not to prove a pending to allow a non-judicial officer, unarmed
petitioners were violating the Fair Labor violative of the Fourth charge or complaint, but upon which to with judicial process, to demand the
Standards Act, including records relating to Amendment. make one if, in the Administrator’s books and papers of an individual is an
coverage. (Investigative subpoenas) judgment, the facts thus discovered open invitation to abuse of that power.
should justify doing so. Only by confining the subpoena power
Petitioners contend that the enforcement of the exclusively to the judiciary can there be
subpoena would permit the Administrator to any insurance against this corrosion of
conduct a general search into their books, liberty. Liberty is too priceless to be
records and papers, in order to secure evidence forfeited through the zeal of an
that they have violated the Act, without prior administrative agent
charge or complaint.

On appeal appellant makes the contention that


compulsory obedience to the investigatory
subpoena constitutes an unreasonable search
and seizure, unless the court first adjudicates
the question of coverage, and that the summary
proceedings pursuant to which the trial court
ordered obedience to the subpoena did not
afford it an opportunity to have the question of
coverage determined in a full and complete
proceedings conducted for that purpose.

Camara v. Municipal Court On November 6, 1963, an inspector of Housing WON administrative Yes. US SC hold that administrative searches Fourth Amendment provides: The right of
Inspection Division of the San Francisco Department searches without warrant is of the kind at issue here are significant the people to be secured in their persons,
Kats of Public Health entered an apartment building to violative of the Fourth intrusions upon the interests protected by houses, papers, and effects, against
make a routine annual inspection for possible Amendment. the Fourth Amendment. Such searches unreasonable searches and seizures, shall
violations of the city's Housing Code. The building's when authorized and conducted without a not be violated, and no Warrants shall
manager informed the inspector that appellant, warrant procedure lack the traditional issue, but upon probable cause, supported
lessee of the ground floor, was using the rear of his safeguards which the Fourth Amendment by Oath or affirmation, and particularly
leasehold as a personal residence. Claiming that the guarantees to the individual.The reasons describing the place to be searched, and
building's occupancy permit did not allow stated in Frank v. State of Maryland and in the persons or things to be seized.
residential use of the ground floor, the inspector other cases for upholding warrantless
confronted appellant. Inspector demanded that searches are insufficient to justify the Purpose: to safeguard the privacy and
appellant permit an inspection of the premises. substantial weakening of the Fourth security of individuals against arbitrary
Appellant refused to allow the inspection because Amendment's protections. The Frank invasions of government officials
the inspector lacked a search warrant. majority gave recognition to the unique
character of these inspection programs by Governing principle: a search of private
The inspector returned on November 8, again refusing to require search warrants; to property without proper consent is
without a warrant. Appellant again refused to allow reject that disposition does not justify unreasonable unless it has been authorized
an inspection. A citation was then mailed ordering ignoring the question whether some other by a valid search warrant
appellant to appear at the district attorney's office. accommodation between public need and
When appellant failed to appear, two inspectors individual rights is essential. Frank v. Maryland: The court upheld the
returned to his apartment on November 22. They conviction of one who refused to permit a
informed appellant that he was required by law to warrantless inspection of private premises
permit an inspection under Section 503 of the for the purpose of locating and abating a
Housing Code. Appellant nevertheless refused the suspected public nuisance.
inspectors access to his apartment without a search
warrant.

Thereafter, a complaint was filed charging him with


refusing to permit a lawful inspection in violation of
Section 507 of the Code. Appellant was arrested on
December 2 and released on bail. When his
demurrer to the criminal complaint was denied,
appellant filed this petition for a writ of prohibition.

Harvey (Spectre) v. Petitioners Andrew Harvey and John Sherman, 52 W/N Search and Seizure Yes, the arrest of petitioners was based on Moncado vs. People's Court, 80 Phil. 1
Defensor-Santiago and 72 years, respectively, are both American was valid. probable cause determined after close [1948].
nationals residing at Pagsanjan, Laguna, while surveillance for three (3) months during
Bryce Adriaan Van Elshout, 58 years old, is a Dutch citizen which period their activities were There can be no question that the right
also residing at Pagsanjan, Laguna. monitored. The existence of probable cause against unreasonable searches and seizures
justified the arrest and the seizure of the guaranteed by Article III, Section 2 of the
The case stems from the apprehension of photo negatives, photographs and posters 1987 Constitution, is available to all
petitioners on 27 February 1988 from their without warrant. Those articles were seized persons, including aliens, whether accused
respective residences by agents of the Commission as an incident to a lawful arrest and, are of crime or not.
on Immigration and Deportation (CID) by virtue of therefore, admissible in evidence.
Mission Orders issued by respondent Commissioner People vs. Syjuco 64 Phil. 667 [1937];
Miriam Defensor Santiago of the CID. Petitioners are Alverez vs. CFI, 64 Phil. 33 [1937]
presently detained at the CID Detention Center.
Such facts and circumstances antecedent to
Petitioners were among the twenty-two (22) the issuance of the warrant that in
suspected alien pedophiles who were apprehended themselves are sufficient to induce a
after three months of close surveillance by CID cautious man to rely on them and act in
agents in Pagsanjan, Laguna. Two (2) days after pursuance thereof.
apprehension, seventeen (17) of the twenty-two
(22) arrested aliens opted for self-deportation and
have left the country. One was released for lack of
evidence; another was charged not for being a
pedophile but for working without a valid working
visa. Thus, of the original twenty two (22), only the
three petitioners have chosen to face deportation.

Seized during petitioner’s apprehension were rolls


of photo negatives and photos of the suspected
child prostitutes shown in salacious poses as well as
boys and girls engaged in the sex act. There were
also posters and other literature advertising the
child prostitutes.

On 7 March 1988, Warrants of Arrest were issued


by respondent against petitioners for violation of
Sections 37, 45 and 46 of the Immigration Act and
Section 69 of the Revised Administrative Code On
the same date, the Board of Special Inquiry III
commenced trial.

On 14 March 1988, petitioners filed an Urgent


Petition for Release Under Bond alleging that their
health was being seriously affected by their
continuous detention. Respondent ordered the CID
doctor to examine petitioners, who certified that
petitioners were healthy.

On 22 March 1988, petitioners filed a Petition for


Bail which, which was denied considering the
certification by the CID physician that petitioners
were healthy. To avoid congestion, respondent
ordered petitioners' transfer to the CID detention
cell at Fort Bonifacio, but the transfer was deferred
pending trial due to the difficulty of transporting
them to and from the CID where trial was on-going.

On 4 April 1988 petitioner Andrew Harvey filed a


Manifestation/Motion stating that he had "finally
agreed to a self-deportation" and praying that he be
"provisionally released for at least 15 days and
placed under the custody of Atty. Asinas before he
voluntarily departs the country." On 7 April 1988,
the Board of Special Inquiry — III allowed
provisional release of five (5) days only under
certain conditions. However, it appears that on the
same date that the aforesaid Manifestation/ Motion
was filed, Harvey and his co-petitioners had already
filed the present petition.

On 4 April 1988, as heretofore stated, petitioners


availed of this Petition for a Writ of Habeas Corpus.
A Return of the Writ was filed by the Solicitor
General and the Court heard the case on oral
argument on 20 April 1988. A Traverse to the Writ
was presented by petitioners to which a Reply was
filed by the Solicitor General.

Petitioners question the validity of their detention


on the following grounds:

1) There is no provision in the Philippine


Immigration Act of 1940 nor under Section 69 of the
Revised Administrative Code, which legally clothes
the Commissioner with any authority to arrest and
detain petitioners pending determination of the
existence of a probable cause leading to an
administrative investigation.
2) Respondent violated Section 2, Article III of the
1987 Constitution prohibiting unreasonable
searches and seizures since the CID agents were
not clothed with valid Warrants of arrest, search
and seizure as required by the said provision.

3) Mere confidential information made to the CID


agents and their suspicion of the activities of
petitioners that they are pedophiles, coupled with
their association with other suspected pedophiles,
are not valid legal grounds for their arrest and
detention unless they are caught in the act. They
further allege that being a pedophile is not
punishable by any Philippine Law nor is it a crime to
be a pedophile.

People v. Aminnudin From the Case Book: Whether or not the NO. The Court ruled that the warrantless The following are instances where a
marijuana leaves may be search and seizure were illegal and warrantless search is allowed:
Mar Idel Aminnudin was arrested shortly after admitted as evidence in any therefore such was inadmissible as 1. A warrantless search
disembarking from the M/V Wilcon 9 in Iloilo City proceeding in order to evidence. And without the evidence of the incidental to a lawful arrest;
when an informer identified him as the carrier of convict him of the crime marijuana leaves allegedly seized from 2. Search of evidence in “plain
marijuana. PC officers who arrested him inspected charged. Aminnudin, the case against him must fall view”;
his bag and found what looked like marijuana leaves for he enjoys the constitutional 3. Search of a moving vehicle;
which were sent to the NBI laboratory for presumption of innocence and he will be so 4. Consented* warrantless
examination. When it was verified that it was declared even if his defense is weak as long search;
indeed marijuana leaves, an information for as the prosecution is not strong enough to 5. Customs search;
violation of the Dangerous Drugs Act was filed convict him. 6. A “stop and frisk” search;
against him. and
The Court found that from the conflicting 7. Exigent and emergency
The admittance of the marijuana leaves as evidence declarations of the PC witnesses, it was circumstances.
against him was questioned in the proceeding for clear that they had at least two (2) days
he was searched and the articles were seized within which they could have obtained a *While it is not to say that Aminnudin is
without warrant as he was disembarking from M/V warrant to arrest and search Aminnudin innocent, for indeed his very words suggest
Wilcon 9. who was coming to Iloilo on the M/V that he is lying, that fact alone does not
Wilcon 9. His name was known. The vehicle justify a finding that he is guilty. The
was identified. The date of his arrival was constitutional presumption is that he is
certain. And from the information received, innocent. The warrantless search and arrest
they could have persuaded a judge that did not come under the exceptions allowed
there was probable cause to justify the by the Rules of Court. Hence, the
issuance of a warrant. Yet they did nothing warrantless search was also illegal and the
to comply with the law and relied on the PC evidence obtained thereby was
Lieutenant’s authority that “a search inadmissible.
warrant was not necessary.”

Warrantless search could also not be


effected for Aminnudin was not caught in
flagrante nor was a crime about to be
committed. There was also no expediency
that could be invoked in order to dispense
with the obtention of the warrant. The
search was also not an incident of a lawful
arrest.

People v. Burgos

Jill

Umil v. Ramos Eight petitions for habeas corpus were consolidated W/N the arrests are valid? NO. Petitioners were arrested legally and Rules of Court Rule 113, Sec. 5:
in this case. They claim that their detention is W/N the petitioners are were not deprived of their constitutional Arrest without warrant; when lawful:
Katrine unlawful since their arrests were made without entitled to the privilege of right to liberty, hence the cases do not a) when the person is caught in flagranti
warrant and that no preliminary investigation was Habeas Corpus? warrant their release on habeas corpus. delicto / in the act of committing an offense
first conducted, hence the informations filed against Warrantless arrests are recognized by law. b) when an offense has just been
them are null and void.The arrests relied on the Evidence does that persons arrested herein committed and the person making the
“confidential information” that the authorities had all freshly committed or were actually arrest has personal knowledge of the facts
received. Except for one case where inciting to committing an offense and so the arrests indicating that the person arrested has
sedition was charged, the rest are charged with were justified. committed it.
subversion for being a member of the New People’s
Army. In the case of Roque v. De Villa, Roque’s Subversion and Rebellion are anchored on
arrest was justified because as officers an ideological base which compels the
The respondents, meanwhile, assert that the and/or members of the NUFC-CPP, their repetition of the same acts of lawlessness
privilege of habeas corpus is not available to the arrest, without warrant, was justified for and violence until the overriding objective
petitioners since they have been legally arrested the same reasons why Rolando Dural was of overthrowing government is attained.
and are detained by the virtue of valid informations arrested - that at the time of apprehension, Hence, a rebel can be arrested anytime,
filed in court. they were in possession of ammunitions even without warrant.
without license to possess them.
Dissenting Opinion, Sarmiento: The
Also, regarding the subversion cases, the “confidential information” was nothing but
arrests were legal since subversion is a form hearsay. The searches and arrests made
of a continuing crime - together with were bereft of probable cause and that the
rebellion, conspiracy or proposal to commit petitioners were not caught in flagrante
rebellion/subversion, and crimes delicto or in any overt act. Utmost, the
committed in furtherance thereof or in authorities was lucky in their fishing
connection therewith. On the inciting to expeditions.
sedition case, the arrest was legal since an
information was filed prior to his arrest. IMPORTANT: The constitutional proscription
Lastly, the arrests were not fishing against unlawful searches and seizures
expeditions but a result of an in-depth therefore applies as a restraint directed
surveillance of NPA safe houses pinpointed only against the government and its
by non other than members of the NPA. agencies tasked with the enforcement of
the law. Thus, it could only be invoked
The right to preliminary investigation against the State to whom the restraint
should be exercised by the offender as soon against arbitrary and unreasonable exercise
as possible. Otherwise, it would be of power is imposed.
considered as impliedly waived and the
filing of information can proceed. This sort
of irregularity is not sufficient to set aside a
valid judgement upon a sufficient complaint
and after a trial free from error.

Go v. CA From the Case Book: Issues: There was no lawful warrantless arrest of Sec 5. Arrest without warrant; when lawful.
petitioner within the maning of Sec 5 of A peach officer or a private person may,
RC Acoording to the findings of the San Juan Police in 1. W/N a lawful warrantless Rule 113. without warrant, arrest a person:
their investigation Report, on 2 July 1991, Eldon arrest had been effected by
Maguannwas driving his car along Wilson St., San the San Juan Police in Petitioner’s arrest took place six days after a. when, in his presence, the person to be
Juan, Metro Manila, heading towards P. Guevarra St. respect of petitioner Go the shooting of Maguan. The “arresting arrested has committed, is actualy
(Sec 5 of Rule 113); officers” obviosuly are not present, within committing or is attempting to commit an
Petitioner entered Wilson St. where it is a one-way the meaning of Section 5a, at the time offense.
street and started traveling in the opposite or 2. W/N petitioner had petitioner had alledged shot Maguan.
“wrong” direction. At the corner of Wilson and J. effectively waived his right b. when an offense has in fact just been
Abad Santos Sts., Petitioner’s and Maguan’s cars to preliminary investigation. Neither could the “arrest” effected six days commited, and he had personal knowledge
nearly bumped each other. Petitioner alighted from after the shooting be reasonably regarded of facts indicating that the person to be
his car, walked over and shit Maguan inside the car. as effected “when [the shooting had] in fact arrested has committed it; and
Petitioner then boarded his car and left the scene. just been committed” within the meaning
of Sec 5b. Moreover, none of the arresting c. when the person to be arrested is a
A security guard at a near by restaurant was able to officers had any personal knowledge of the prisoner who has escaped while being
take down petitioner’s car plate number. The police facts indicating the petitioner was the transferred from one confiement to
arrived shortly thereafter at the scene of the gunman who shot Maguan. another.
shooting and there retrieved an empty shell and
one round of live ammunition for a 9 mm caliber --- In cases falling under paragraphs a and b
pistol. Verification at LTO showed that the car was Umil vs Ramos was cited by both the hereof, the person arrested without a
registered to one Elsa Ang Go. petitioner and the Solicitor general. It was warrant shall be forth delivered to the
an eight-to-six vote, the court sustained the nearest police station or jail and she shall
The following day, the police retured to the scene of legality of the warrantless arrest of be proceed against in accordance with Rule
the shooting to find out where the suspect had petitioners made from 1 to 14 days after 112, Sec 7.
come from; they were informed that petitioner had the actual commission of the offense, upon
dined at Cravings Bake Shop shortly before the ground that such offenses constituted
shooting. The police obtained a facsimile or “continuing crimes” – usually commited by
impression of the credit card used by petitioner NPAs (subversion, etc).
from the cashier of the bake shop. The security
guard was shown a picture of the petitioner which In the instant case, the offense of the
he identified as the same person. petitioner was murder, commenced and
completed on one location and this is not a
On July 8, 1991, petitioner presented himself before “continuing crime.”
San Juan Police Station to verify news reports that
he was being hunted by the police; he was Warrantless “arrest” in this case does not
accompanied by 2 lawyers. The police forthwith fall within the terms of Section 5 of Rule
detained him. An eyewitness to the shooting who 113 of the 1985 Rules on Criminal
was at the police station at the time positively Procedure.
identified petitioner as the gunman.

People v. Mengote From book: WON the arrest and search No. The requirements of rule 115 Section 5 Rule 113, Section 5, of the Rules of Court:
of Mengote and the seizure of the rules of court is not established and
Kats Before noon of August 8, 1987, after the Western of the revolver from him therefore their arrest without warrant is Sec. 5. Arrest without warrant when
Police District received a telephone call from an were lawful. invalid. Evidence obtained as a result of an lawful. — A peace officer or private person
informer that there were three suspicious-looking illegal search or seizure is inadmissible in may, without a warrant, arrest a person;
persons at the corner of Juan Luna and North Bay any proceeding for any purpose. This is the
Boulevard in Tondo, Manila. A surveillance team of absolute prohibition of Article II Sec 2 of (a) When, in his presence, the person to be
plainclothesmen was forthwith dispatched to the the Constitution. arrested has committed, is actually
place. There they saw two men "looking from side committing, or is attempting to commit an
to side," one of whom was holding his abdomen. offense;
They approached these persons and identified
themselves as policemen, whereupon the two tried (b) When an offense has in fact just been
to run away but were unable to escape because the committed, and he has personal knowledge
other lawmen had surrounded them. The suspects of facts indicating that the person to be
were then searched. One of them, who turned out arrested has committed it; and
to be the accused-appellant, was found with a .38
caliber Smith and Wesson revolver with six live (c) When the person to be arrested is a
bullets in the chamber. His companion, later prisoner who has escaped from a penal
identified as Nicanor Morellos, had a fan knife establishment or place where he is serving
secreted in his front right pants pocket. The final judgment or temporarily confined
weapons were taken from them. Mengote and while his case is pending, or has escaped
Morellos were then turned over to police while being transferred from one
headquarters for investigation by the Intelligence confinement to another.
Division.
In cases falling under paragraphs (a) and (b)
Accused-appellant Rogelio Mengote was convicted hereof, the person arrested without a
of illegal possession of firearms on the strength warrant shall be forthwith delivered to the
mainly of the stolen pistol found on his person at nearest police station or jail, and he shall be
the moment of his warrantless arrest. In this appeal, proceeded against in accordance with Rule
he pleads that the weapon was not admissible as 112, Section 7.
evidence against him because it had been illegally
seized and was therefore the fruit of the poisonous People v. Malmstedt: the Court sustained
tree. The Government disagrees. It insists that the the warrantless arrest of the accused
revolver was validly received in evidence by the trial because there was a bulge in his waist that
judge because its seizure was incidental to an arrest excited the suspicion of the arresting officer
that was doubtless lawful even if admittedly and, upon inspection, turned out to be a
without warrant. pouch containing hashish

People v. Claudio: the accused boarded a


bus and placed the buri bag she was
carrying behind the seat of the arresting
officer while she herself sat in the seat
before him. His suspicion aroused, be
surreptitiously examined the bag, which he
found to contain marijuana. He then and
there made the warrantless arrest and
seizure that we subsequently upheld on the
ground that probable cause had been
sufficiently established

People v. Aminnudin: The Court held that


the warrantless arrest of the accused was
unconstitutional. This was effected while be
was coming down a vessel, to all
appearances no less innocent than the
other disembarking passengers. He had not
committed nor was be actually committing
or attempting to commit an offense in the
presence of the arresting officers. He was
not even acting suspiciously. In short, there
was no probable cause that, as the
prosecution incorrectly suggested,
dispensed with the constitutional
requirement of a warrant.

Manalili v. CA At about 2:10 oclock in the afternoon of April 11, Whether or not the NO. The evidence is legally admissible.We The Constitutional prohibition against
1988, policemen from the Anti-Narcotics Unit of the evidence is a product of hold that the search was valid, being akin to unreasonable arrests, searches and seizures
Jen Kalookan City Police Station were conducting a illegal search and is a stop-and-frisk. A stop-and-frisk was refers to those effected without a validly
surveillance along A. Mabini street, Kalookan City, in therefore inadmissible defined as the vernacular designation of issued warrant, subject to certain
front of the Kalookan City Cemetery. The policemen the right of a police officer to stop a citizen exceptions:
were Pat. Romeo Espiritu and Pat. Anger Lumabas on the street, interrogate him, and pat him
and a driver named Arnold Enriquez was driving a for weapon(s). (1) customs searches;
Tamaraw vehicle which was the official car of the (2) search of moving vehicles;
Police Station of Kalookan City. The surveillance was The general rule is that a search and seizure (3) seizure of evidence in plain view;
being made because of information that drug must be validated by a previously secured (4) consent searches;
addicts were roaming the area in front of the judicial warrant; otherwise, such search and (5) a search incidental to a lawful arrest;
Kalookan City Cemetery. seizure is unconstitutional and subject to and
challenge. This right, however, is not (6) a “stop and frisk.”
Upon reaching the Kalookan City Cemetery, the absolute. Stop-and-frisk has already been
policemen alighted from their vehicle. They then adopted as another exception to the In the landmark case of Terry vs. Ohio, a
chanced upon a male person in front of the general rule against a search without a stop-and-frisk was defined as the
cemetery who appeared high on drugs. The male warrant. In the case at hand, Patrolman vernacular designation of the right of a
person was observed to have reddish eyes and to be Espiritu and his companions observed police officer to stop a citizen on the
walking in a swaying manner. When this male during their surveillance that appellant had street, interrogate him, and pat him for
person tried to avoid the policemen, the latter red eyes and was wobbling like a drunk weapon(s):
approached him and introduced themselves as along the Caloocan City Cemetery, which
police officers. The policemen then asked the male according to police information was a x x x (W)here a police officer observes an
person what he was holding in his hands. The male popular hangout of drug addicts. From his unusual conduct which leads him
person tried to resist. Pat. Romeo Espiritu asked the experience as a member of the reasonably to conclude in light of his
male person if he could see what said male person Anti-Narcotics Unit of the Caloocan City experience that criminal activity may be
had in his hands. The latter showed the wallet and Police, such suspicious behavior was afoot and that the persons with whom he is
allowed Pat. Romeo Espiritu to examine the same. characteristic of drug addicts who were dealing may be armed and presently
Pat. Espiritu took the wallet and examined it. He high. The policemen therefore had dangerous, where in the course of
found suspected crushed marijuana residue inside. sufficient reason to stop petitioner to investigating this behavior he identified
He kept the wallet and its marijuana contents. investigate if he was actually high on drugs. himself as a policeman and makes
During such investigation, they found reasonable inquiries, and where nothing in
The male person was then brought to the marijuana in petitioner’s possession the initial stages of the encounter serves to
Anti-Narcotics Unit of the Kalookan City Police dispel his reasonable fear for his own or
Headquarters and was turned over to Cpl. Wilfredo others safety, he is entitled for the
Tamondong for investigation. Pat. Espiritu also protection of himself and others in the area
turned over to Cpl. Tamondong the confiscated to conduct a carefully limited search of the
wallet and its suspected marijuana contents. The outer clothing of such persons in an
man turned out to be the accused ALAIN MANALILI attempt to discover weapons which might
y DIZON. be used to assault him. Such a search is a
reasonable search under the Fourth
The RTC convicted petitioner of illegal possession of Amendment, and any weapon seized may
marijuana residue largely on the strength of the properly be introduced in evidence against
arresting officers testimony. CA affirmed RTC’s the person from whom they were taken.
decision.

Potrebbero piacerti anche